COC REVISION MCQFiI (1).pptx for exam prepation

nagesageshu6 2 views 238 slides Oct 02, 2025
Slide 1
Slide 1 of 450
Slide 1
1
Slide 2
2
Slide 3
3
Slide 4
4
Slide 5
5
Slide 6
6
Slide 7
7
Slide 8
8
Slide 9
9
Slide 10
10
Slide 11
11
Slide 12
12
Slide 13
13
Slide 14
14
Slide 15
15
Slide 16
16
Slide 17
17
Slide 18
18
Slide 19
19
Slide 20
20
Slide 21
21
Slide 22
22
Slide 23
23
Slide 24
24
Slide 25
25
Slide 26
26
Slide 27
27
Slide 28
28
Slide 29
29
Slide 30
30
Slide 31
31
Slide 32
32
Slide 33
33
Slide 34
34
Slide 35
35
Slide 36
36
Slide 37
37
Slide 38
38
Slide 39
39
Slide 40
40
Slide 41
41
Slide 42
42
Slide 43
43
Slide 44
44
Slide 45
45
Slide 46
46
Slide 47
47
Slide 48
48
Slide 49
49
Slide 50
50
Slide 51
51
Slide 52
52
Slide 53
53
Slide 54
54
Slide 55
55
Slide 56
56
Slide 57
57
Slide 58
58
Slide 59
59
Slide 60
60
Slide 61
61
Slide 62
62
Slide 63
63
Slide 64
64
Slide 65
65
Slide 66
66
Slide 67
67
Slide 68
68
Slide 69
69
Slide 70
70
Slide 71
71
Slide 72
72
Slide 73
73
Slide 74
74
Slide 75
75
Slide 76
76
Slide 77
77
Slide 78
78
Slide 79
79
Slide 80
80
Slide 81
81
Slide 82
82
Slide 83
83
Slide 84
84
Slide 85
85
Slide 86
86
Slide 87
87
Slide 88
88
Slide 89
89
Slide 90
90
Slide 91
91
Slide 92
92
Slide 93
93
Slide 94
94
Slide 95
95
Slide 96
96
Slide 97
97
Slide 98
98
Slide 99
99
Slide 100
100
Slide 101
101
Slide 102
102
Slide 103
103
Slide 104
104
Slide 105
105
Slide 106
106
Slide 107
107
Slide 108
108
Slide 109
109
Slide 110
110
Slide 111
111
Slide 112
112
Slide 113
113
Slide 114
114
Slide 115
115
Slide 116
116
Slide 117
117
Slide 118
118
Slide 119
119
Slide 120
120
Slide 121
121
Slide 122
122
Slide 123
123
Slide 124
124
Slide 125
125
Slide 126
126
Slide 127
127
Slide 128
128
Slide 129
129
Slide 130
130
Slide 131
131
Slide 132
132
Slide 133
133
Slide 134
134
Slide 135
135
Slide 136
136
Slide 137
137
Slide 138
138
Slide 139
139
Slide 140
140
Slide 141
141
Slide 142
142
Slide 143
143
Slide 144
144
Slide 145
145
Slide 146
146
Slide 147
147
Slide 148
148
Slide 149
149
Slide 150
150
Slide 151
151
Slide 152
152
Slide 153
153
Slide 154
154
Slide 155
155
Slide 156
156
Slide 157
157
Slide 158
158
Slide 159
159
Slide 160
160
Slide 161
161
Slide 162
162
Slide 163
163
Slide 164
164
Slide 165
165
Slide 166
166
Slide 167
167
Slide 168
168
Slide 169
169
Slide 170
170
Slide 171
171
Slide 172
172
Slide 173
173
Slide 174
174
Slide 175
175
Slide 176
176
Slide 177
177
Slide 178
178
Slide 179
179
Slide 180
180
Slide 181
181
Slide 182
182
Slide 183
183
Slide 184
184
Slide 185
185
Slide 186
186
Slide 187
187
Slide 188
188
Slide 189
189
Slide 190
190
Slide 191
191
Slide 192
192
Slide 193
193
Slide 194
194
Slide 195
195
Slide 196
196
Slide 197
197
Slide 198
198
Slide 199
199
Slide 200
200
Slide 201
201
Slide 202
202
Slide 203
203
Slide 204
204
Slide 205
205
Slide 206
206
Slide 207
207
Slide 208
208
Slide 209
209
Slide 210
210
Slide 211
211
Slide 212
212
Slide 213
213
Slide 214
214
Slide 215
215
Slide 216
216
Slide 217
217
Slide 218
218
Slide 219
219
Slide 220
220
Slide 221
221
Slide 222
222
Slide 223
223
Slide 224
224
Slide 225
225
Slide 226
226
Slide 227
227
Slide 228
228
Slide 229
229
Slide 230
230
Slide 231
231
Slide 232
232
Slide 233
233
Slide 234
234
Slide 235
235
Slide 236
236
Slide 237
237
Slide 238
238
Slide 239
239
Slide 240
240
Slide 241
241
Slide 242
242
Slide 243
243
Slide 244
244
Slide 245
245
Slide 246
246
Slide 247
247
Slide 248
248
Slide 249
249
Slide 250
250
Slide 251
251
Slide 252
252
Slide 253
253
Slide 254
254
Slide 255
255
Slide 256
256
Slide 257
257
Slide 258
258
Slide 259
259
Slide 260
260
Slide 261
261
Slide 262
262
Slide 263
263
Slide 264
264
Slide 265
265
Slide 266
266
Slide 267
267
Slide 268
268
Slide 269
269
Slide 270
270
Slide 271
271
Slide 272
272
Slide 273
273
Slide 274
274
Slide 275
275
Slide 276
276
Slide 277
277
Slide 278
278
Slide 279
279
Slide 280
280
Slide 281
281
Slide 282
282
Slide 283
283
Slide 284
284
Slide 285
285
Slide 286
286
Slide 287
287
Slide 288
288
Slide 289
289
Slide 290
290
Slide 291
291
Slide 292
292
Slide 293
293
Slide 294
294
Slide 295
295
Slide 296
296
Slide 297
297
Slide 298
298
Slide 299
299
Slide 300
300
Slide 301
301
Slide 302
302
Slide 303
303
Slide 304
304
Slide 305
305
Slide 306
306
Slide 307
307
Slide 308
308
Slide 309
309
Slide 310
310
Slide 311
311
Slide 312
312
Slide 313
313
Slide 314
314
Slide 315
315
Slide 316
316
Slide 317
317
Slide 318
318
Slide 319
319
Slide 320
320
Slide 321
321
Slide 322
322
Slide 323
323
Slide 324
324
Slide 325
325
Slide 326
326
Slide 327
327
Slide 328
328
Slide 329
329
Slide 330
330
Slide 331
331
Slide 332
332
Slide 333
333
Slide 334
334
Slide 335
335
Slide 336
336
Slide 337
337
Slide 338
338
Slide 339
339
Slide 340
340
Slide 341
341
Slide 342
342
Slide 343
343
Slide 344
344
Slide 345
345
Slide 346
346
Slide 347
347
Slide 348
348
Slide 349
349
Slide 350
350
Slide 351
351
Slide 352
352
Slide 353
353
Slide 354
354
Slide 355
355
Slide 356
356
Slide 357
357
Slide 358
358
Slide 359
359
Slide 360
360
Slide 361
361
Slide 362
362
Slide 363
363
Slide 364
364
Slide 365
365
Slide 366
366
Slide 367
367
Slide 368
368
Slide 369
369
Slide 370
370
Slide 371
371
Slide 372
372
Slide 373
373
Slide 374
374
Slide 375
375
Slide 376
376
Slide 377
377
Slide 378
378
Slide 379
379
Slide 380
380
Slide 381
381
Slide 382
382
Slide 383
383
Slide 384
384
Slide 385
385
Slide 386
386
Slide 387
387
Slide 388
388
Slide 389
389
Slide 390
390
Slide 391
391
Slide 392
392
Slide 393
393
Slide 394
394
Slide 395
395
Slide 396
396
Slide 397
397
Slide 398
398
Slide 399
399
Slide 400
400
Slide 401
401
Slide 402
402
Slide 403
403
Slide 404
404
Slide 405
405
Slide 406
406
Slide 407
407
Slide 408
408
Slide 409
409
Slide 410
410
Slide 411
411
Slide 412
412
Slide 413
413
Slide 414
414
Slide 415
415
Slide 416
416
Slide 417
417
Slide 418
418
Slide 419
419
Slide 420
420
Slide 421
421
Slide 422
422
Slide 423
423
Slide 424
424
Slide 425
425
Slide 426
426
Slide 427
427
Slide 428
428
Slide 429
429
Slide 430
430
Slide 431
431
Slide 432
432
Slide 433
433
Slide 434
434
Slide 435
435
Slide 436
436
Slide 437
437
Slide 438
438
Slide 439
439
Slide 440
440
Slide 441
441
Slide 442
442
Slide 443
443
Slide 444
444
Slide 445
445
Slide 446
446
Slide 447
447
Slide 448
448
Slide 449
449
Slide 450
450

About This Presentation

Read each question


Slide Content

INTERNAL MEDICINE and PEDIATRICS MCQ FOR HO prep. by Dr. Fekadu .T 8/27/2021 1

PART -I INTERNAL MEDICINE prep. by Dr. Fekadu .T 8/27/2021 2

Q-1 1-A 19-year-old college student presented with episodic attacks of shortness of breath,dry cough and wheezing of 5 years duration. These symptoms usually occur at times of weather changes or whenever she catches cold, and tend to worsen during night and early morning. The patient has a history of recurrent sneezing, runny nose and itchy eyes of the same duration. she is also allergic to egg. The most likely diagnosis? A . B.Asthma B.lung abscess C . Chronic bronchitis D.COPD prep. by Dr. Fekadu .T 8/27/2021 3

Answer-A B asthma Is a syndrom xzd by air flow obstruction that varies both spontaneously and with tx COPD Chronic bronchitis - a clinically defined as a condition with chronic cough and phlegm /thick mucus/ at least 3 months in a year for over 2 consecutive years . Pulmonary Emphysema - an anatomically defined as a condition characterized by destruction and enlargement of the lung alveoli prep. by Dr. Fekadu .T 8/27/2021 4

lung abscess is defined as collection of pus within a destroyed portion of the lung. It may develop following necrotizing infections of the lung (bacterial pneumonia, TB, fungi) Commonly caused by pyogenic bacteria:- S. aureus, Klebsiella , mixed anaerobes, and Nocardia . prep. by Dr. Fekadu .T 8/27/2021 5

Q-2 2-A 15 years old boy who is diabetic for the past four years present with abdominal pain,vomiting and shortness of breathing .He has history of fever and sore throat two days back.On investigation his random blood sugar 350mg/ dl,CBC on normal range, U/A Kenton + 3. The most likely diagnosis is ? A.Diabetic Ketoacidosis B . Infection C.Renal failure D.UTI prep. by Dr. Fekadu .T 8/27/2021 6

Q-3 3-A 69-year-old male patient with 45 pack years smoking history presented with worsening of shortness of breath and wheezing of 5 days duration. The patient had a history of productive cough for the past 6 years. He stated all his symptoms were exacerbated when he smoked cigarette or when he got the flu. On p/exam his vital /s were within normal range, oxygen saturation was found to be 93% on ambient air, barrel chest with hyper-resonant note on percussion was appreciated and chest auscultation revealed bilateral polymorphic wheeze and prolonged expiration. The most likely diagnosis? A.C OPD B. B.Asthma C . lung abscess D.Community acquered pneumonia prep. by Dr. Fekadu .T 8/27/2021 7

Q-4 4-A 15-year-old healthy boy presented with hematuria ,swelling of the lower extrimities of 1 week duration.He has history of pharyngitis on physical exam vital sign BP 150/70mmhg ,RR 18/min PR 120b/min Temp 37 0c he has Oedema Urinalysis reveals 3+ proteinuria. RBC casts with Dysmorphic red blood cells. Which of the following is the most likely diagnosis? Nephrotic syndrome B . Congestive heart failure C . Minimal change disease D . Poststreptococcal glomerulonephritis prep. by Dr. Fekadu .T 8/27/2021 8

Q-5 5-34-A 45-year-old woman presents with fever and cough. She has had no past medical problems and was well until about 3 days ago.She has also dyspnea,pleuritic chest pain and Mucopurulent sputum production.Physical examination is remarkable for a temperature of 390C, O2saturation is 85% and the presence of diffuse rales on chest examination. Except for an elevated white count with a left-shifted differential, her blood tests are normal.Chest X- Ray reveals bilateral infiltrates.The most likely diagnosis? A.SEVERE community-acquired pne umoni a B . Lung abscess C.Hospital -acquired pneumonia D . Bronchiectasis prep. by Dr. Fekadu .T 8/27/2021 9

Q-6 6-For this patient in question (no ).The most common potential microbiologic etiology is/? A.S.pneumo niae B, Chlamydia pneumonia C.Staphylococcusaureus , D.Mycoplasma pneumonia prep. by Dr. Fekadu .T 8/27/2021 10

Q-7 7.The 45-year-old women presented with shortness of breath and paroxysmal nocturnal dyspnea.On physical exam she has S3 gallop, rales and bilateral lower extremity gradeIII edema .Chest x-ray shows cardiomegaly, bilateral infiltrates, and cephalization.The most likely diagnosis? A.Congestive heart failure B . Chronic renal failure C -Hypertension D . Chronic liver disease prep. by Dr. Fekadu .T 8/27/2021 11

Q-8 8-A 27-year-old college freshman complains of dysuria and urinary frequency of three days durations. She has also fever, chills, or flank pain, Urinalysis reveals 8 to 10 white blood cells (WBCs) per high-power field and numerous gram-negative bacteria. The most likely diagnosis is? A.chronic urinary tract infection B.pyelonephritis C.Cystitis D. Urethritis prep. by Dr. Fekadu .T 8/27/2021 12

Q-9 9-An 18-yearold woman presents 2 weeks after an exudative pharyngitis with fever and arthritis that is asymmetrical and involves more than three joints. The arthritis is migratory , affecting one joint for several day s and imp roving , then affecting another joint. On physical exam, The patient has a macular rash consisting of ring -shaped lesions with clear centers and her cardiac examination reveal s an S3 gallop. Which of the following is the most likely diagnosis? A. Gout B. Endocarditis C. Rheumatoid arthritis D. Rheumaic fever prep. by Dr. Fekadu .T 8/27/2021 13

Q-10 10-A 33 years old female patient from Chencha is admitted to E ward after presenting with a compliant of reddish discoloration of urine of 2 days duration. In addition to this she has also a Blood pressure record of 150/100mmHg and bilateral pitting leg edema. Urinalysis showed many dysmorphic RBs and proteinuria of + 3.Her creatinine level is 5.4mg/dl. 4 months back she had a creatinine measurement of 1.1mg/dl. What is the most likely diagnosis? A-Nephrotic Syndrome B-Acute glomeluronephritis C-Acute pyelonephritis D-CKD prep. by Dr. Fekadu .T 8/27/2021 14

The nephrotic syndrome is a clinical complex characterized by : Proteinuria >3.5g/day Hypoalbuminemia Edema Hyperlipidemia Hypercoagulability prep. by Dr. Fekadu .T 8/27/2021 15

Acute nephritic syndrome is characterised by Sudden onset of: Hematuria Oliguria (< 500 ml/day, or < 0.5 ml/kg/h) Edema Hypertension Proteinuria non nephrotic range prep. by Dr. Fekadu .T 8/27/2021 16

Chronic Renal failure: - progressive and irreversible reduction of the renal function, over a period of more than 6 months , to a level less than 20 % of the normal, as a result of destruction of significant number of nephrons. prep. by Dr. Fekadu .T 8/27/2021 17

Pyelonephritis is an inflammation of the kidney Symptoms of acute pyelonephritis generally develop rapidly over a few hours or a day and include a fever, shaking chills, nausea, vomiting Symptoms of cystitis may or may not be present. Physical examination: fever, tachycardia, and generalized muscle tenderness, Costvertebral angle tenderness prep. by Dr. Fekadu .T 8/27/2021 18

Q-11 11-Abeba is known asthmatic for the past 06 years, presented with compliant of shortness of breath which doesn’t respond for Salbutamol.She has history of repeated hospital admission. On examination RR=54, SO 2 =81%. She uses her accessory muscles and she cannot able to speak. Chest is hyper-resonant on auscultation.Which one of the following is not true regarding management of this patient ? A.Oxygen , 2–4 L/min nasal cannula or 40–60% by mask B.Salbutamol 6 puff every 20 min for 01 hr C.IV Hydrocortisone 100mg QID D.Ceftriaxon 01 grm IV BID with Azythromycin 500 mg PO daily prep. by Dr. Fekadu .T 8/27/2021 19

12-A 35 year old male athlete with heart failure is confortable at rest and during walking nearby . He develops dysnea and fatigue when he runs more than 10m distance .what is the stage and functional class of this patient ? Stage C Class II Stage C Class I Stage B Class III Stage B Class II Stage B Class I prep. by Dr. Fekadu .T 8/27/2021 20

prep. by Dr. Fekadu .T 8/27/2021 21

…………..mgt HF should be viewed as a continuum that is comprised of four interrelated stages. Stage A : patients who are at high risk for developing HF but without structural heart disease or symptoms of HF (e.g., DM, HPN). Stage B : patients who have structural heart disease but without symptoms of HF (e.g., previous MI and asymptomatic LV dysfunction). Stage C patients who have structural heart disease and have developed symptoms of HF ( e.g., patients with a previous MI with dyspnea and fatigue ). Stage D patients with refractory HF requiring special interventions ( e.g., pts with refractory HF who are awaiting cardiac transplantation ). prep. by Dr. Fekadu .T 8/27/2021 22

13.A 45 year old male known hypertensive patient for past 12 year presented with compliant of SOB ,night cough and PND of 01 month .physical examination revealed tender hepatomegaly and cardiothoracic ratio of 60 % . He has no pertinent finding on respiratory system and his ejection fraction is 65% . What is the most likely diagnosis? A. Systolic heart failure B. Diastolic heart failure C. Corpulmonale D. All prep. by Dr. Fekadu .T 8/27/2021 23

14-A known CLD patient from konso present with a compliant of abdominal pain, fever and on physical examination abdominal tenderness; peritoneal fluid analysis reveals WBC=670cells/mm3.The diagnosis considered as Spontaneous Bacterial Peritonitis, the most likely etiologic agent? A.S.aureus , B.Enterococcus sp. C .E.Coli D.Tuberculosis   prep. by Dr. Fekadu .T 8/27/2021 24

15-Which antibiotics are the first line treatments for the above diagnosis? Ceftriaxone Gentamycin Azithromycin Amoxicillin prep. by Dr. Fekadu .T 8/27/2021 25

17-A 5 year old female child presented with generalized body swelling of 1wk duration. She has history of skin rash a month back. And has hx of cola colored urine of 3 days duration. On P/E- V/S: PR: 110, RR: 28, T-37, BP: 140/80 and Urine analysis reveals full of RBC..The most likely diagnosis of this patient is: A.Nephrotic syndrome B.post streptococcal glomerulonephritis C.disseminated TB D.none prep. by Dr. Fekadu .T 8/27/2021 26

18-A 26 year old male patient presented with headache, fever and neck pain. On physical examination he has a temperature of 39.2 degree celcius and all meningeal signs are positive. All of the following findings of CSF analysis suggest pyogenic meningitis EXCEPT : A.A CSF WBC count of 10,000 per microliter B.A CSF glucose concentration of 28mg/dl C.A CSF protein concentration of 20mg/dl D.An intracellular gram negative diplococcic on gram stain prep. by Dr. Fekadu .T 8/27/2021 27

Diagnosis of bacterial meningitis is made by CSF examination CSF parameter Typical finding in bacterial meningitis Opening pressure 200-500mmH2O White blood cell count 10-10,000/µl; %Neutrophils >80% RBC Absent in non-traumatic tap Protein >45 mg/dl; 100-500mg/dl Glucose <40mg/dl; CSF:serum glucose<0.4 Gram Stain Positive in 60-90% Culture Positive in 70-85% prep. by Dr. Fekadu .T 8/27/2021 28

19-A 50-year-old woman has complained of pain and swelling in her proximal interphalangeal joints, both wrists and knees. She complains of morning stiffness. She has had a hysterectomy 10 yea rs ago. Physical exam shows swelling and thickening of the PIP joints.Hemoglobin is 10.3 g/ dL , MCV 80 fl , serum iron 8 μmol /L, ironbinding capacity 40 μmol /L (normal: 45 to 66), saturation 20%. The most likely explanation for this woman’s anemia is A.Occult blood loss B.Vitamin deficiency C.Anemia of chronic disease D.Sideroblastic anemia prep. by Dr. Fekadu .T 8/27/2021 29

20-A 70-year-old intensive care unit patient complains of fever and shaking chills. The patient develops hypotension, and blood cultures are positive for gram-negative bacilli. The patient begins bleeding from venipuncture sites and around his Foley catheter. Hct : 38% WBC : 15.00 _ 103 mm ,Platelet count: 40,000 per mm3 (normal: 130,000 to 400,000) Peripheral blood smear: fragmented RBCs PT: elevated PTT: elevated Plasma fibrinogen: 70 mg/ dL (200 to 400).The best course of therapy in this patient is; A.Begin heparin B.Treat underlying disease C.Begin plasmapheresis D.Give vitamin K prep. by Dr. Fekadu .T 8/27/2021 30

21-A 22 year old female C-II medical student presented EOPD with complaint of fever ,shaking chills of 04 day duration. She has non- productive cough and pleuritic chest pain of the same duration . she has no significant past medical history . physical examination revealed crepitation on the posterior lower chest bilaterally . Which one is the best initial investigation to confirm the diagnosis ? A.Gene expert B.Gram stain of sputum C.Chest X –ray D.culture of sputum prep. by Dr. Fekadu .T 8/27/2021 31

22-A good indicator of diabetic patient’s disease control over near past is A.Serial blood sugar levels B.Clinical signs and symptoms C.Hemoglobin A 1 C level D.All of the above prep. by Dr. Fekadu .T 8/27/2021 32

23-Abebe is a 35 year old farmer .He presented with bilateral leg swelling of a month duration. On examination he has raised JVP and diastolic murmur at the apex .which of the following drug is important to delay disease progression? A ) Spironolactone B ) Enalapril C) Digoxin D) Aspirin prep. by Dr. Fekadu .T 8/27/2021 33

24-A 30 year- old prisoner presents with intermittent fever and headache of two days duration . He claims that five of his room Mate had similar symptoms and where treated in the prleon . blood film examination showed a splrochets which one of the following is most appropriate antibiotics choice for the pt ? A, ceftriaxone B,ciproflocacillin C, procaine penicillin D , vancomycin prep. by Dr. Fekadu .T 8/27/2021 34

Relapsing fever A common illness in refugee & military camps, among prisoners, street children and people living in crowded environment Relapsing fever is caused by Borrelia species , which are spirochetal gram negative helical bacteria. Louse borne /Endemic relapsing fever: Anthroponotic infection, caused only by B. recurrentis & transmitted by body lice Tick borne / Epidemic relapsing fever: Zoonotic infection, caused by variety of Borrelia species ( B. duttonii , B. hermsii , B. turicatae , B . crocidurae , B. hispanica ) & transmitted by soft ticks. Blood film is the gold standard dxic modality with >70% yield. D ark-field microscopy - to see motile spirochetes TX Give P rocaine Penicillin G 600,000 I.M stat’ (Adult) For children: 200,000-400,000units . 6-Continue R x with Doxycycline 100mg PO BID for 5days. prep. by Dr. Fekadu .T 8/27/2021 35

25-A 25 year- old male with known liver disease presents with caugh , fever and weight loss for one month sputum examination for acid fast bacilli is positive 3 times. Which combination anti-tuberculosis treatment is most appropriate for this patient? A, 2ERHZ/4HE B, 2RHZ/4RH C , 2SERH/6RH prep. by Dr. Fekadu .T 8/27/2021 36

26-A 55 year- old male comes to emergency OPD because of excruciating pain in his right big toe.He was diagnosed to have gouty arthritis. Which of the following is the best intial pharmacotheraphy ? A, Ceftriaxone B , Allopurinol C , indomethacin D,Morphine prep. by Dr. Fekadu .T 8/27/2021 37

27-A health officer treating a 74 years old man who was admitted with shortness of breathing and fever . his tempreture on admission was 39 degrec.on physical examination he is sick looking and has decrease air entery at the right basal lung field , the chest x-ray shows an infiltrate at the right lower lobe. What is the most likely causative organism? A, streptococcus pneumonia B,Legionella pneumophila C,viral pneumonia D, klebsiella pneumonia prep. by Dr. Fekadu .T 8/27/2021 38

28-A 35 year- old pregnant woman presents with fever easly fatigability and change in mental status. On physical examination , BP=100/60mmhg, PR=108/min, RR=20/ min.To =37.5oC and has pale conjunctiva and confused. Laboratory examination reveals hemoglobin=7g/dl, random blood sugar=30mg/dl and plasmodium falciparum positive . which of the following in the initial step in the management of this patient ? A, Artesunate IV B, Artemether-lumefantrine orally C, Blood transfusion D, Glucose infusion prep. by Dr. Fekadu .T 8/27/2021 39

29-A 22 years old HIV positive man with a presumptive diagnosis of pneummocystis jiroveci pneumonia hasno signs of respiratory distress. Which of the following is the most appropriate pharmacotherapy? A, Fluconazole B, fluconazole and prednisolone C, Cotrimoxazole D,Cotrimoxazole and prednisolone prep. by Dr. Fekadu .T 8/27/2021 40

Pneumocystis jiroveci , formerly known as pneumocystis carini pneumonia commonly occurs when patients have significant immune suppression ( CD4<200cells/mm3 or CD4 percentage < 14%). Clinical /m low grade fever, dry cough, fatigue and progressive dyspnea exacerbated by exertion . INV Chest X-ray: presumptive diagnosis of PCP is based on clinical judgment and typical chest Xray findings revealing a perihilar interstitial infiltration with tendency to spread outwards. Note that the chest X-ray can be normal in 20% of patients . Culture: definitive diagnosis of PCP First line: Trimethoprim + Sulphamethoxazole for 3 weeks*COTRI Trimethoprim+Sulphamethoxazole , 15-20mg/kg/day prep. by Dr. Fekadu .T 8/27/2021 41

30-A 47 year old man comes to the hospital complaining of epigastric pain that is exacerbated by eating. A test for helicopacter pylori turned positive and endoscopy confirmed a duodenal ulcer. Which one is the most appropriatetherapy ? A.amoxicillin,clarithromycin and omeprazole B.metronidazole and anti acids C.omeprazole and antiacids D.rantidine,amoxicillin and metronidazole prep. by Dr. Fekadu .T 8/27/2021 42

H. Pylori positive: H. pylori eradiaction therapyFirst line therapy All drugs for 7-14 days Amoxicillin , 1gm, P.O. BID - Clarithromycin, 500mg, P. O., BID PLUS PPI Alternative (for penicillin allergic patients ). This regimen has a higher failure rate . All drugs for 7-14 days - Clarithromycin, 500mg P.O. BID PLUS - Metronidazole, 500mg, P.O. BID PLUS - PPI prep. by Dr. Fekadu .T 8/27/2021 43

Answer-A B asthma Is a syndrom xzd by air flow obstruction that varies both spontaneously and with tx COPD Chronic bronchitis - a clinically defined as a condition with chronic cough and phlegm /thick mucus/ at least 3 months in a year for over 2 consecutive years . Pulmonary Emphysema - an anatomically defined as a condition characterized by destruction and enlargement of the lung alveoli prep. by Dr. Fekadu .T 8/27/2021 44

lung abscess is defined as collection of pus within a destroyed portion of the lung. It may develop following necrotizing infections of the lung (bacterial pneumonia, TB, fungi) Commonly caused by pyogenic bacteria:- S. aureus, Klebsiella , mixed anaerobes, and Nocardia . prep. by Dr. Fekadu .T 8/27/2021 45

Q-2 2-A 15 years old boy who is diabetic for the past four years present with abdominal pain,vomiting and shortness of breathing .He has history of fever and sore throat two days back.On investigation his random blood sugar 350mg/ dl,CBC on normal range, U/A Kenton + 3. The most likely diagnosis is ? A.Diabetic Ketoacidosis B . Infection C.Renal failure D.UTI prep. by Dr. Fekadu .T 8/27/2021 46

Q-3 3-A 69-year-old male patient with 45 pack years smoking history presented with worsening of shortness of breath and wheezing of 5 days duration. The patient had a history of productive cough for the past 6 years. He stated all his symptoms were exacerbated when he smoked cigarette or when he got the flu. On p/exam his vital /s were within normal range, oxygen saturation was found to be 93% on ambient air, barrel chest with hyper-resonant note on percussion was appreciated and chest auscultation revealed bilateral polymorphic wheeze and prolonged expiration. The most likely diagnosis? A.C OPD B. B.Asthma C . lung abscess D.Community acquered pneumonia prep. by Dr. Fekadu .T 8/27/2021 47

Q-4 4-A 15-year-old healthy boy presented with hematuria ,swelling of the lower extrimities of 1 week duration.He has history of pharyngitis on physical exam vital sign BP 150/70mmhg ,RR 18/min PR 120b/min Temp 37 0c he has Oedema Urinalysis reveals 3+ proteinuria. RBC casts with Dysmorphic red blood cells. Which of the following is the most likely diagnosis? Nephrotic syndrome B . Congestive heart failure C . Minimal change disease D . Poststreptococcal glomerulonephritis prep. by Dr. Fekadu .T 8/27/2021 48

Q-5 5-34-A 45-year-old woman presents with fever and cough. She has had no past medical problems and was well until about 3 days ago.She has also dyspnea,pleuritic chest pain and Mucopurulent sputum production.Physical examination is remarkable for a temperature of 390C, O2saturation is 85% and the presence of diffuse rales on chest examination. Except for an elevated white count with a left-shifted differential, her blood tests are normal.Chest X- Ray reveals bilateral infiltrates.The most likely diagnosis? A.SEVERE community-acquired pne umoni a B . Lung abscess C.Hospital -acquired pneumonia D . Bronchiectasis prep. by Dr. Fekadu .T 8/27/2021 49

Q-6 6-For this patient in question (no ).The most common potential microbiologic etiology is/? A.S.pneumo niae B, Chlamydia pneumonia C.Staphylococcusaureus , D.Mycoplasma pneumonia prep. by Dr. Fekadu .T 8/27/2021 50

Q-7 7.The 45-year-old women presented with shortness of breath and paroxysmal nocturnal dyspnea.On physical exam she has S3 gallop, rales and bilateral lower extremity gradeIII edema .Chest x-ray shows cardiomegaly, bilateral infiltrates, and cephalization.The most likely diagnosis? A.Congestive heart failure B . Chronic renal failure C -Hypertension D . Chronic liver disease prep. by Dr. Fekadu .T 8/27/2021 51

Q-8 8-A 27-year-old college freshman complains of dysuria and urinary frequency of three days durations. She has also fever, chills, or flank pain, Urinalysis reveals 8 to 10 white blood cells (WBCs) per high-power field and numerous gram-negative bacteria. The most likely diagnosis is? A.chronic urinary tract infection B.pyelonephritis C.Cystitis D. Urethritis prep. by Dr. Fekadu .T 8/27/2021 52

Q-9 9-An 18-yearold woman presents 2 weeks after an exudative pharyngitis with fever and arthritis that is asymmetrical and involves more than three joints. The arthritis is migratory , affecting one joint for several day s and imp roving , then affecting another joint. On physical exam, The patient has a macular rash consisting of ring -shaped lesions with clear centers and her cardiac examination reveal s an S3 gallop. Which of the following is the most likely diagnosis? A. Gout B. Endocarditis C. Rheumatoid arthritis D. Rheumaic fever prep. by Dr. Fekadu .T 8/27/2021 53

Q-10 10-A 33 years old female patient from Chencha is admitted to E ward after presenting with a compliant of reddish discoloration of urine of 2 days duration. In addition to this she has also a Blood pressure record of 150/100mmHg and bilateral pitting leg edema. Urinalysis showed many dysmorphic RBs and proteinuria of + 3.Her creatinine level is 5.4mg/dl. 4 months back she had a creatinine measurement of 1.1mg/dl. What is the most likely diagnosis? A-Nephrotic Syndrome B-Acute glomeluronephritis C-Acute pyelonephritis D-CKD prep. by Dr. Fekadu .T 8/27/2021 54

The nephrotic syndrome is a clinical complex characterized by : Proteinuria >3.5g/day Hypoalbuminemia Edema Hyperlipidemia Hypercoagulability prep. by Dr. Fekadu .T 8/27/2021 55

Acute nephritic syndrome is characterised by Sudden onset of: Hematuria Oliguria (< 500 ml/day, or < 0.5 ml/kg/h) Edema Hypertension Proteinuria non nephrotic range prep. by Dr. Fekadu .T 8/27/2021 56

Chronic Renal failure: - progressive and irreversible reduction of the renal function, over a period of more than 6 months , to a level less than 20 % of the normal, as a result of destruction of significant number of nephrons. prep. by Dr. Fekadu .T 8/27/2021 57

Pyelonephritis is an inflammation of the kidney Symptoms of acute pyelonephritis generally develop rapidly over a few hours or a day and include a fever, shaking chills, nausea, vomiting Symptoms of cystitis may or may not be present. Physical examination: fever, tachycardia, and generalized muscle tenderness, Costvertebral angle tenderness prep. by Dr. Fekadu .T 8/27/2021 58

Q-11 11-Abeba is known asthmatic for the past 06 years, presented with compliant of shortness of breath which doesn’t respond for Salbutamol.She has history of repeated hospital admission. On examination RR=54, SO 2 =81%. She uses her accessory muscles and she cannot able to speak. Chest is hyper-resonant on auscultation.Which one of the following is not true regarding management of this patient ? A.Oxygen , 2–4 L/min nasal cannula or 40–60% by mask B.Salbutamol 6 puff every 20 min for 01 hr C.IV Hydrocortisone 100mg QID D.Ceftriaxon 01 grm IV BID with Azythromycin 500 mg PO daily prep. by Dr. Fekadu .T 8/27/2021 59

12-A 35 year old male athlete with heart failure is confortable at rest and during walking nearby . He develops dysnea and fatigue when he runs more than 10m distance .what is the stage and functional class of this patient ? Stage C Class II Stage C Class I Stage B Class III Stage B Class II Stage B Class I prep. by Dr. Fekadu .T 8/27/2021 60

prep. by Dr. Fekadu .T 8/27/2021 61

…………..mgt HF should be viewed as a continuum that is comprised of four interrelated stages. Stage A : patients who are at high risk for developing HF but without structural heart disease or symptoms of HF (e.g., DM, HPN). Stage B : patients who have structural heart disease but without symptoms of HF (e.g., previous MI and asymptomatic LV dysfunction). Stage C patients who have structural heart disease and have developed symptoms of HF ( e.g., patients with a previous MI with dyspnea and fatigue ). Stage D patients with refractory HF requiring special interventions ( e.g., pts with refractory HF who are awaiting cardiac transplantation ). prep. by Dr. Fekadu .T 8/27/2021 62

13.A 45 year old male known hypertensive patient for past 12 year presented with compliant of SOB ,night cough and PND of 01 month .physical examination revealed tender hepatomegaly and cardiothoracic ratio of 60 % . He has no pertinent finding on respiratory system and his ejection fraction is 65% . What is the most likely diagnosis? A. Systolic heart failure B. Diastolic heart failure C. Corpulmonale D. All prep. by Dr. Fekadu .T 8/27/2021 63

14-A known CLD patient from konso present with a compliant of abdominal pain, fever and on physical examination abdominal tenderness; peritoneal fluid analysis reveals WBC=670cells/mm3.The diagnosis considered as Spontaneous Bacterial Peritonitis, the most likely etiologic agent? A.S.aureus , B.Enterococcus sp. C .E.Coli D.Tuberculosis   prep. by Dr. Fekadu .T 8/27/2021 64

15-Which antibiotics are the first line treatments for the above diagnosis? Ceftriaxone Gentamycin Azithromycin Amoxicillin prep. by Dr. Fekadu .T 8/27/2021 65

17-A 5 year old female child presented with generalized body swelling of 1wk duration. She has history of skin rash a month back. And has hx of cola colored urine of 3 days duration. On P/E- V/S: PR: 110, RR: 28, T-37, BP: 140/80 and Urine analysis reveals full of RBC..The most likely diagnosis of this patient is: A.Nephrotic syndrome B.post streptococcal glomerulonephritis C.disseminated TB D.none prep. by Dr. Fekadu .T 8/27/2021 66

18-A 26 year old male patient presented with headache, fever and neck pain. On physical examination he has a temperature of 39.2 degree celcius and all meningeal signs are positive. All of the following findings of CSF analysis suggest pyogenic meningitis EXCEPT : A.A CSF WBC count of 10,000 per microliter B.A CSF glucose concentration of 28mg/dl C.A CSF protein concentration of 20mg/dl D.An intracellular gram negative diplococcic on gram stain prep. by Dr. Fekadu .T 8/27/2021 67

Diagnosis of bacterial meningitis is made by CSF examination CSF parameter Typical finding in bacterial meningitis Opening pressure 200-500mmH2O White blood cell count 10-10,000/µl; %Neutrophils >80% RBC Absent in non-traumatic tap Protein >45 mg/dl; 100-500mg/dl Glucose <40mg/dl; CSF:serum glucose<0.4 Gram Stain Positive in 60-90% Culture Positive in 70-85% prep. by Dr. Fekadu .T 8/27/2021 68

19-A 50-year-old woman has complained of pain and swelling in her proximal interphalangeal joints, both wrists and knees. She complains of morning stiffness. She has had a hysterectomy 10 yea rs ago. Physical exam shows swelling and thickening of the PIP joints.Hemoglobin is 10.3 g/ dL , MCV 80 fl , serum iron 8 μmol /L, ironbinding capacity 40 μmol /L (normal: 45 to 66), saturation 20%. The most likely explanation for this woman’s anemia is A.Occult blood loss B.Vitamin deficiency C.Anemia of chronic disease D.Sideroblastic anemia prep. by Dr. Fekadu .T 8/27/2021 69

20-A 70-year-old intensive care unit patient complains of fever and shaking chills. The patient develops hypotension, and blood cultures are positive for gram-negative bacilli. The patient begins bleeding from venipuncture sites and around his Foley catheter. Hct : 38% WBC : 15.00 _ 103 mm ,Platelet count: 40,000 per mm3 (normal: 130,000 to 400,000) Peripheral blood smear: fragmented RBCs PT: elevated PTT: elevated Plasma fibrinogen: 70 mg/ dL (200 to 400).The best course of therapy in this patient is; A.Begin heparin B.Treat underlying disease C.Begin plasmapheresis D.Give vitamin K prep. by Dr. Fekadu .T 8/27/2021 70

21-A 22 year old female C-II medical student presented EOPD with complaint of fever ,shaking chills of 04 day duration. She has non- productive cough and pleuritic chest pain of the same duration . she has no significant past medical history . physical examination revealed crepitation on the posterior lower chest bilaterally . Which one is the best initial investigation to confirm the diagnosis ? A.Gene expert B.Gram stain of sputum C.Chest X –ray D.culture of sputum prep. by Dr. Fekadu .T 8/27/2021 71

22-A good indicator of diabetic patient’s disease control over near past is A.Serial blood sugar levels B.Clinical signs and symptoms C.Hemoglobin A 1 C level D.All of the above prep. by Dr. Fekadu .T 8/27/2021 72

23-Abebe is a 35 year old farmer .He presented with bilateral leg swelling of a month duration. On examination he has raised JVP and diastolic murmur at the apex .which of the following drug is important to delay disease progression? A ) Spironolactone B ) Enalapril C) Digoxin D) Aspirin prep. by Dr. Fekadu .T 8/27/2021 73

24-A 30 year- old prisoner presents with intermittent fever and headache of two days duration . He claims that five of his room Mate had similar symptoms and where treated in the prleon . blood film examination showed a splrochets which one of the following is most appropriate antibiotics choice for the pt ? A, ceftriaxone B,ciproflocacillin C, procaine penicillin D , vancomycin prep. by Dr. Fekadu .T 8/27/2021 74

Relapsing fever A common illness in refugee & military camps, among prisoners, street children and people living in crowded environment Relapsing fever is caused by Borrelia species , which are spirochetal gram negative helical bacteria. Louse borne /Endemic relapsing fever: Anthroponotic infection, caused only by B. recurrentis & transmitted by body lice Tick borne / Epidemic relapsing fever: Zoonotic infection, caused by variety of Borrelia species ( B. duttonii , B. hermsii , B. turicatae , B . crocidurae , B. hispanica ) & transmitted by soft ticks. Blood film is the gold standard dxic modality with >70% yield. D ark-field microscopy - to see motile spirochetes TX Give P rocaine Penicillin G 600,000 I.M stat’ (Adult) For children: 200,000-400,000units . 6-Continue R x with Doxycycline 100mg PO BID for 5days. prep. by Dr. Fekadu .T 8/27/2021 75

25-A 25 year- old male with known liver disease presents with caugh , fever and weight loss for one month sputum examination for acid fast bacilli is positive 3 times. Which combination anti-tuberculosis treatment is most appropriate for this patient? A, 2ERHZ/4HE B, 2RHZ/4RH C , 2SERH/6RH prep. by Dr. Fekadu .T 8/27/2021 76

26-A 55 year- old male comes to emergency OPD because of excruciating pain in his right big toe.He was diagnosed to have gouty arthritis. Which of the following is the best intial pharmacotheraphy ? A, Ceftriaxone B , Allopurinol C , indomethacin D,Morphine prep. by Dr. Fekadu .T 8/27/2021 77

27-A health officer treating a 74 years old man who was admitted with shortness of breathing and fever . his tempreture on admission was 39 degrec.on physical examination he is sick looking and has decrease air entery at the right basal lung field , the chest x-ray shows an infiltrate at the right lower lobe. What is the most likely causative organism? A, streptococcus pneumonia B,Legionella pneumophila C,viral pneumonia D, klebsiella pneumonia prep. by Dr. Fekadu .T 8/27/2021 78

28-A 35 year- old pregnant woman presents with fever easly fatigability and change in mental status. On physical examination , BP=100/60mmhg, PR=108/min, RR=20/ min.To =37.5oC and has pale conjunctiva and confused. Laboratory examination reveals hemoglobin=7g/dl, random blood sugar=30mg/dl and plasmodium falciparum positive . which of the following in the initial step in the management of this patient ? A, Artesunate IV B, Artemether-lumefantrine orally C, Blood transfusion D, Glucose infusion prep. by Dr. Fekadu .T 8/27/2021 79

29-A 22 years old HIV positive man with a presumptive diagnosis of pneummocystis jiroveci pneumonia hasno signs of respiratory distress. Which of the following is the most appropriate pharmacotherapy? A, Fluconazole B, fluconazole and prednisolone C, Cotrimoxazole D,Cotrimoxazole and prednisolone prep. by Dr. Fekadu .T 8/27/2021 80

Pneumocystis jiroveci , formerly known as pneumocystis carini pneumonia commonly occurs when patients have significant immune suppression ( CD4<200cells/mm3 or CD4 percentage < 14%). Clinical /m low grade fever, dry cough, fatigue and progressive dyspnea exacerbated by exertion . INV Chest X-ray: presumptive diagnosis of PCP is based on clinical judgment and typical chest Xray findings revealing a perihilar interstitial infiltration with tendency to spread outwards. Note that the chest X-ray can be normal in 20% of patients . Culture: definitive diagnosis of PCP First line: Trimethoprim + Sulphamethoxazole for 3 weeks*COTRI Trimethoprim+Sulphamethoxazole , 15-20mg/kg/day prep. by Dr. Fekadu .T 8/27/2021 81

30-A 47 year old man comes to the hospital complaining of epigastric pain that is exacerbated by eating. A test for helicopacter pylori turned positive and endoscopy confirmed a duodenal ulcer. Which one is the most appropriatetherapy ? A.amoxicillin,clarithromycin and omeprazole B.metronidazole and anti acids C.omeprazole and antiacids D.rantidine,amoxicillin and metronidazole prep. by Dr. Fekadu .T 8/27/2021 82

H. Pylori positive: H. pylori eradiaction therapyFirst line therapy All drugs for 7-14 days Amoxicillin , 1gm, P.O. BID - Clarithromycin, 500mg, P. O., BID PLUS PPI Alternative (for penicillin allergic patients ). This regimen has a higher failure rate . All drugs for 7-14 days - Clarithromycin, 500mg P.O. BID PLUS - Metronidazole, 500mg, P.O. BID PLUS - PPI prep. by Dr. Fekadu .T 8/27/2021 83

Peditrics -I questns For ho -I prep. by Dr. Fekadu .T 8/27/2021 84

1-A 2 year old male child presented with cough, high grade fever and coryza associated with this he has skin rash that progressed from head to trunk and redness of the eye of 3 days duration.On P/E- V/S- PR-144 RR-40 T-37.8 Integ – multiple maculo papular rashes over the face and trunk. All of the following are false about the above problem, except? A.The most likely diagnosis is chickenpox B.Mostly caused by toxic bacteria C.It is not contagious D.Infection confers life-long immunity prep. by Dr. Fekadu .T 8/27/2021 85

2-A 12 years old female child presented with high grade fever of one day duration associated with this she has loss of appetite, vomiting and flank pain. Two days back she had pain and frequency of urination. The most likely diagnosis is? A.Malaria B.Cystitis C.Pyelonephriti s D.All prep. by Dr. Fekadu .T 8/27/2021 86

3-A 5 year old female child presented with generalized body swelling of 1wk duration. She has history of skin rash a month back. And has hx of cola colored urine of 3 days duration. On P/E- V/S: PR: 110, RR: 28, T-37, BP: 140/80 and Urine analysis reveals full of RBC. The most likely diagnosis of this patient is: A.Nephrotic syndrome B.post streptococcal glomerulonephritis C.disseminated TB D.CLD prep. by Dr. Fekadu .T 8/27/2021 87

4-If you encounter a neonate with a scaphoid abdomen, having respiratory distress and upon auscultation there is bowel sound heard on the left side of the chest. What could be the possible diagnosis for this neonate? A.Chonal atresia B.Diaphragmatic hernia C.Tracheoesophageal fistula D.None of the above prep. by Dr. Fekadu .T 8/27/2021 88

4-Among the different vaccine which is available in our country which vaccine is protective and has a good efficiency in preventing tuberculosis? A.PCV B.BCG C.OPV D.Pentavalent prep. by Dr. Fekadu .T 8/27/2021 89

5-Which of the following is not a contraindication to do lumbar puncture in patient you suspected neonatal meningitis? A.Bulging fontanel B.Focal neurologic deficit C.Thrombocytopenia D.Infection at the site of LP E.None of the above prep. by Dr. Fekadu .T 8/27/2021 90

6-A 7 yrs old male patient presented to you at pediatric emergency OPD complaining of generalized body swelling of week duration. Additionally, he has history low grade fever and decreased urine amount, severe headache and one episode of abnormal body movement but, he denies of having urine color change. Two weeks back he had history of sore throat at which it resolved out spontaneously without treatment.On P/Ex he has puffy face, V/S: BP 150/100mmHg PR: 110 bpm RR : 28 T: 37.6 He has grade II bilateral pitting edemaOn U/A there is microscopic hematuria (dysmorphic RBCs) Which one is the most likely diagnosis and it’s feared complication for this patient? A.Nephrotic syndrome- uremic encephalopathy B.Severe acute malnutrition- hypoglycemia C.Nephtitic syndrome- hypertensive encephalopathy D.Acute rheumatic fever- rheumatic heart disease prep. by Dr. Fekadu .T 8/27/2021 91

7-A 10 years old female known asthmatic patient whom on follow up presented to emergency OPD with dyspnea, cough but no fever. On examination v/s: PR: 110 RR: 50 T: 37.4 oC and the pulse oximetry reads 88% of O2 saturation in room air. She has sign of distress and diffuse wheezing over the whole chest and no other pertinent finding. What would be your next best stepyou should follow in the of management this patient? A.Send her immediately for CXR B.Put on her Oxygen therapy and start salbutamol challenge C.Hold on any treatment and call for anesthesiologist for endotracheal intubation D.Provide her IV antibiotics prep. by Dr. Fekadu .T 8/27/2021 92

8-One of the following is not true about CSF finding in normal child A.CSF glucose < 75% of blood glucose B.CSF protein 20-45g/dl C.cell count 0-5/ microlitter D.opening pressure of 50-85mmhg prep. by Dr. Fekadu .T 8/27/2021 93

9-A child is found to have feeble pulse, cold extremities and oliguria of 1day and diarrhoea and vomiting of 5days; what is the 1 st action to be taken? A/Secure Iv line and give 2oml/kg of blood B/Give steroid C/Secure Iv line and give 2oml/kg of normal saline D/Subcutaneous adrenaline 10.A 13 month old child presented to the health center with no immunization at all before; which vaccines should be given? A-Pentavalent , measles, OPV1 B-Pentavalent , measles C-Pentavalent , OPV1 D-measles , OPV1 prep. by Dr. Fekadu .T 8/27/2021 94

11-A 2yrs old 10 kg child is assessed as having some dehydration because he is irritable and has sunken eyes. How do you manage him? A-ORS 1000ml over 5hrs B-RL 750ml over 5hrs C-ORS 750ml over 4hrs D-RL 1000ml over 4hrs 12-Which one of the following is not true? A- all patient with sever pneumonia need admission B-for patient with foreign body aspiration bronchodilators are indicated C- chronic Otitis media is ear discharge for 14 days or more D-pneumonia can present only with fast breathing and cough prep. by Dr. Fekadu .T 8/27/2021 95

13-A 3yrs old child sustained a 2 nd scald burn of 20 %; his body surface area is 0.5 and his weight is 12kg; what is the amount of fluid requirement? A-1000ml B-960ml C-860ml D-320ml prep. by Dr. Fekadu .T 8/27/2021 96

14-One of the following is not a feature of edematous malnutrition : A-Generalized edema B-Dermatosis C-Ascites D-Significant loss of subcutaneous tissue prep. by Dr. Fekadu .T 8/27/2021 97

15-A significant number of under-five children were diagnosed with serious neurological problems and anemia in a certain district. Researchers found out that the families of these children were strict vegetarians. Which nutritional deficiency are the children suffering from? A. Folic Acid B. Vitamin B12/ C. Vitamin B3 D. Vitamin B6 prep. by Dr. Fekadu .T 8/27/2021 98

16-A 4 day female neonate is brought to health centere with failure to open her mouth and breast feed . She was well in the first three days after which the mather noticed excessive cry with stiff body posturing . The mather had no ANC follow up and delivery was at home.what is the most likely diagnosis in this neonate? A. Hypocalcemia B. sepsis C. Meningitis D. Tetanus\ prep. by Dr. Fekadu .T 8/27/2021 99

17-A 3year old boy came to the OPD with diarrhea.Stoole examination revealed trophozoite of Gardia lambelia . What is the first line drug to treat this parasite? A. Tinidazole B. Metronidazole C. Paromomycin D. Albendazole prep. by Dr. Fekadu .T 8/27/2021 100

18-which of the following micro-nutrients enhances the absorption of iron? A. Calcium B. Vitamin E C. Vitamin C / D. Zink prep. by Dr. Fekadu .T 8/27/2021 101

19-A health officer was attending delivery of a post term pregnancy. Upon follow-up of the lobar there was a record of fetal tachycardia and grade III meconium stained amniotic fluid. The newborn then failed to breathe spontaneously immediately after delivery. What is the most likely cause of respiratory problem in this baby ? A. Respiratory distress syndrome B. Congenital pneumonia C. Meconium aspiration syndrome / D. Congenital heart disease prep. by Dr. Fekadu .T 8/27/2021 102

20-An 18-months old child with sever acute malnutrition is noticed to have diffuse left thighswelling which is erythematous, hot and tender to touch. It is also fluctuating.which one of the following is the most likely diagnosis ? A. Cellulitis B.Hip arthritis C. Osteomyelitis D. pyomyositis prep. by Dr. Fekadu .T 8/27/2021 103

21-A 3 year-old boy is brought to the ER with high grade fever and shortness of breathof two days. On physical examination RR=62/ min,T = 39oC , lower chest indrawing with bilateral basal coarse creptations . The chest X-ray showed left basal lung pneumatocele . Which of the following is the most likely etiology? A. Haemophilus influenza B. Staphylococcus aureus C. Streptococcus pneumonia D. Streptococcus pyogenes prep. by Dr. Fekadu .T 8/27/2021 104

23-A 7 year old female patient who is on follow-up for bronchial asthma presented with shortness of breath of three hours . On physical examination she is conscious, able to talk in complete sentence, RR=62/ minand PR=110/min. chest examination reveals diffuse expiratory wheeze. Whatis the initial best step in the management of this patient? A. Aminophylline B. Magnesium sulphate C. Oral prednisolone Epinephrine D. sulbutamol puff / prep. by Dr. Fekadu .T 8/27/2021 105

24-A 2 year- old child presented with difficulty of breathing of two hours. He had a breaking cough and hoarseness of voice for the last two days. On physical exam he is agitated,has audible stridor at rest and lower chest indrawing . What is the most likely diagnosis ? A. Croup syndrome B. Foreign body aspiration C. Pertussis D. Severe pneumonia prep. by Dr. Fekadu .T 8/27/2021 106

24-A 3 year-old boy presented with a complaint of intense pruritis worse at night. On examination there are vesicular and popular eruptions at the interdigital spaces. Whichone is the best treatment for thiS child? Dx scabies A. Acyclovir B.Erythromycin C. Mebendazole D. Permethrin/ prep. by Dr. Fekadu .T 8/27/2021 107

25-A 5 year-old boy sustained apenetrating injuryto his right leg . Two weeks later he develop swelling of the injured leg and high fever. X-ray of the affected limb showed features of acute osteomyelitis . What is the most likely etiology in the above case scenario ? A. pseudomonas aeurginosa B. Klebsiella pneumonia C. Streptococcus pyogenes/ D. E.coli prep. by Dr. Fekadu .T 8/27/2021 108

26-A 36 month-old male infant presented with high grade fever and vomiting of two days duration. He has right costovertebral angle tenderness. White cell count showed leukocytosis with left shift and blood film is negative forhemoparasites . What is the most likely diagnosis in this child? A. Pyelonephritis./ B. Hepatitis C. Appendicitis D. Lobar pneumonia prep. by Dr. Fekadu .T 8/27/2021 109

27-A 14 –month old male child is brought to the ERwith watery diarrhea of 24 hours duration. Physical examination reveals irritable infant, weight= 12 kg. sunken eye balls and eagerness to drink. What is the correct type and volume of fluid for this infant ? A. Ringer's lactate,300ml B.0.4% Normal saline, 700ml C. oral rehydration solution, 900ml/ D. Ringer's lactate, 500ml then ORS 500ml prep. by Dr. Fekadu .T 8/27/2021 110

28-which of the following vaccines should NOT be given in an infant with severe immune suppression ? A. Hepatitis B Vaccine B. Measles Vaccine/ C. Meningococcal vaccine prep. by Dr. Fekadu .T 8/27/2021 111

28-An 8- year-old boy known cardiac patient developed lowgrade fevwr , fatigue myalgia in the last two weeks.On examination he was found to have developed a new murmur, petechiael rashes and mild splenomegaly, which one is the most likely diagnosis? A. Acute rheumatic fever B. Dilated cardiomyopathy C. infective endocarditis/ D. Rheumatic heart disease prep. by Dr. Fekadu .T 8/27/2021 112

29-A 9 month old boy is brought to a health centerin a unresponsive state after repeated episodes of vomiting and diarrhea . Physical examination showed a rapide and feeble pulse , skin pich returns very slowely . Multiple attempts to scure an iv line have failed. What is the next best step in the management of this infant? A. Do blood film and determine blood glucose B. Give ORS through NG-tube and refer / C. obtain blood sample for complete blood count D. perform a lumbar puncture prep. by Dr. Fekadu .T 8/27/2021 113

30-A 6 year old diabetic child admitted to a Hospital with a diagnosis of severe diabetic acidosis has been receiving normal saline and insulin for the last three hours. Which electrolyte abnormality is expected? A. hypomagnesemia B. Hyponatremia C.Hypokalmia / D. HYpophosphatemia prep. by Dr. Fekadu .T 8/27/2021 114

Peripheral neuritis (manifesting as paresthesias of the hands and feet) which is the most common adverse effect of isoniazide might be due to deficiency of which vitamin? A-Ascorbic acid B-Niacin C-Pyridoxine D-Folic acid prep. by Dr. Fekadu .T 8/27/2021 115

1-A 2 year old male child presented with cough, high grade fever and coryza associated with this he has skin rash that progressed from head to trunk and redness of the eye of 3 days duration.On P/E- V/S- PR-144 RR-40 T-37.8 Integ – multiple maculo papular rashes over the face and trunk. All of the following are false about the above problem, except? A.The most likely diagnosis is chickenpox B.Mostly caused by toxic bacteria C.It is not contagious D.Infection confers life-long immunity prep. by Dr. Fekadu .T 8/27/2021 116

2-A 12 years old female child presented with high grade fever of one day duration associated with this she has loss of appetite, vomiting and flank pain. Two days back she had pain and frequency of urination. The most likely diagnosis is? A.Malaria B.Cystitis C.Pyelonephriti s D.All prep. by Dr. Fekadu .T 8/27/2021 117

3-A 5 year old female child presented with generalized body swelling of 1wk duration. She has history of skin rash a month back. And has hx of cola colored urine of 3 days duration. On P/E- V/S: PR: 110, RR: 28, T-37, BP: 140/80 and Urine analysis reveals full of RBC. The most likely diagnosis of this patient is: A.Nephrotic syndrome B.post streptococcal glomerulonephritis C.disseminated TB D.CLD prep. by Dr. Fekadu .T 8/27/2021 118

4-If you encounter a neonate with a scaphoid abdomen, having respiratory distress and upon auscultation there is bowel sound heard on the left side of the chest. What could be the possible diagnosis for this neonate? A.Chonal atresia B.Diaphragmatic hernia C.Tracheoesophageal fistula D.None of the above prep. by Dr. Fekadu .T 8/27/2021 119

4-Among the different vaccine which is available in our country which vaccine is protective and has a good efficiency in preventing tuberculosis? A.PCV B.BCG C.OPV D.Pentavalent prep. by Dr. Fekadu .T 8/27/2021 120

5-Which of the following is not a contraindication to do lumbar puncture in patient you suspected neonatal meningitis? A.Bulging fontanel B.Focal neurologic deficit C.Thrombocytopenia D.Infection at the site of LP E.None of the above prep. by Dr. Fekadu .T 8/27/2021 121

6-A 7 yrs old male patient presented to you at pediatric emergency OPD complaining of generalized body swelling of week duration. Additionally, he has history low grade fever and decreased urine amount, severe headache and one episode of abnormal body movement but, he denies of having urine color change. Two weeks back he had history of sore throat at which it resolved out spontaneously without treatment.On P/Ex he has puffy face, V/S: BP 150/100mmHg PR: 110 bpm RR : 28 T: 37.6 He has grade II bilateral pitting edemaOn U/A there is microscopic hematuria (dysmorphic RBCs) Which one is the most likely diagnosis and it’s feared complication for this patient? A.Nephrotic syndrome- uremic encephalopathy B.Severe acute malnutrition- hypoglycemia C.Nephtitic syndrome- hypertensive encephalopathy D.Acute rheumatic fever- rheumatic heart disease prep. by Dr. Fekadu .T 8/27/2021 122

7-A 10 years old female known asthmatic patient whom on follow up presented to emergency OPD with dyspnea, cough but no fever. On examination v/s: PR: 110 RR: 50 T: 37.4 oC and the pulse oximetry reads 88% of O2 saturation in room air. She has sign of distress and diffuse wheezing over the whole chest and no other pertinent finding. What would be your next best stepyou should follow in the of management this patient? A.Send her immediately for CXR B.Put on her Oxygen therapy and start salbutamol challenge C.Hold on any treatment and call for anesthesiologist for endotracheal intubation D.Provide her IV antibiotics prep. by Dr. Fekadu .T 8/27/2021 123

8-One of the following is not true about CSF finding in normal child A.CSF glucose < 75% of blood glucose B.CSF protein 20-45g/dl C.cell count 0-5/ microlitter D.opening pressure of 50-85mmhg prep. by Dr. Fekadu .T 8/27/2021 124

9-A child is found to have feeble pulse, cold extremities and oliguria of 1day and diarrhoea and vomiting of 5days; what is the 1 st action to be taken? A/Secure Iv line and give 2oml/kg of blood B/Give steroid C/Secure Iv line and give 2oml/kg of normal saline D/Subcutaneous adrenaline 10.A 13 month old child presented to the health center with no immunization at all before; which vaccines should be given? A-Pentavalent , measles, OPV1 B-Pentavalent , measles C-Pentavalent , OPV1 D-measles , OPV1 prep. by Dr. Fekadu .T 8/27/2021 125

11-A 2yrs old 10 kg child is assessed as having some dehydration because he is irritable and has sunken eyes. How do you manage him? A-ORS 1000ml over 5hrs B-RL 750ml over 5hrs C-ORS 750ml over 4hrs D-RL 1000ml over 4hrs 12-Which one of the following is not true? A- all patient with sever pneumonia need admission B-for patient with foreign body aspiration bronchodilators are indicated C- chronic Otitis media is ear discharge for 14 days or more D-pneumonia can present only with fast breathing and cough prep. by Dr. Fekadu .T 8/27/2021 126

13-A 3yrs old child sustained a 2 nd scald burn of 20 %; his body surface area is 0.5 and his weight is 12kg; what is the amount of fluid requirement? A-1000ml B-960ml C-860ml D-320ml prep. by Dr. Fekadu .T 8/27/2021 127

14-One of the following is not a feature of edematous malnutrition : A-Generalized edema B-Dermatosis C-Ascites D-Significant loss of subcutaneous tissue prep. by Dr. Fekadu .T 8/27/2021 128

15-A significant number of under-five children were diagnosed with serious neurological problems and anemia in a certain district. Researchers found out that the families of these children were strict vegetarians. Which nutritional deficiency are the children suffering from? A. Folic Acid B. Vitamin B12/ C. Vitamin B3 D. Vitamin B6 prep. by Dr. Fekadu .T 8/27/2021 129

16-A 4 day female neonate is brought to health centere with failure to open her mouth and breast feed . She was well in the first three days after which the mather noticed excessive cry with stiff body posturing . The mather had no ANC follow up and delivery was at home.what is the most likely diagnosis in this neonate? A. Hypocalcemia B. sepsis C. Meningitis D. Tetanus\ prep. by Dr. Fekadu .T 8/27/2021 130

17-A 3year old boy came to the OPD with diarrhea.Stoole examination revealed trophozoite of Gardia lambelia . What is the first line drug to treat this parasite? A. Tinidazole B. Metronidazole C. Paromomycin D. Albendazole prep. by Dr. Fekadu .T 8/27/2021 131

18-which of the following micro-nutrients enhances the absorption of iron? A. Calcium B. Vitamin E C. Vitamin C / D. Zink prep. by Dr. Fekadu .T 8/27/2021 132

19-A health officer was attending delivery of a post term pregnancy. Upon follow-up of the lobar there was a record of fetal tachycardia and grade III meconium stained amniotic fluid. The newborn then failed to breathe spontaneously immediately after delivery. What is the most likely cause of respiratory problem in this baby ? A. Respiratory distress syndrome B. Congenital pneumonia C. Meconium aspiration syndrome / D. Congenital heart disease prep. by Dr. Fekadu .T 8/27/2021 133

20-An 18-months old child with sever acute malnutrition is noticed to have diffuse left thighswelling which is erythematous, hot and tender to touch. It is also fluctuating.which one of the following is the most likely diagnosis ? A. Cellulitis B.Hip arthritis C. Osteomyelitis D. pyomyositis prep. by Dr. Fekadu .T 8/27/2021 134

21-A 3 year-old boy is brought to the ER with high grade fever and shortness of breathof two days. On physical examination RR=62/ min,T = 39oC , lower chest indrawing with bilateral basal coarse creptations . The chest X-ray showed left basal lung pneumatocele . Which of the following is the most likely etiology? A. Haemophilus influenza B. Staphylococcus aureus C. Streptococcus pneumonia D. Streptococcus pyogenes prep. by Dr. Fekadu .T 8/27/2021 135

23-A 7 year old female patient who is on follow-up for bronchial asthma presented with shortness of breath of three hours . On physical examination she is conscious, able to talk in complete sentence, RR=62/ minand PR=110/min. chest examination reveals diffuse expiratory wheeze. Whatis the initial best step in the management of this patient? A. Aminophylline B. Magnesium sulphate C. Oral prednisolone Epinephrine D. sulbutamol puff / prep. by Dr. Fekadu .T 8/27/2021 136

24-A 2 year- old child presented with difficulty of breathing of two hours. He had a breaking cough and hoarseness of voice for the last two days. On physical exam he is agitated,has audible stridor at rest and lower chest indrawing . What is the most likely diagnosis ? A. Croup syndrome B. Foreign body aspiration C. Pertussis D. Severe pneumonia prep. by Dr. Fekadu .T 8/27/2021 137

24-A 3 year-old boy presented with a complaint of intense pruritis worse at night. On examination there are vesicular and popular eruptions at the interdigital spaces. Whichone is the best treatment for thiS child? Dx scabies A. Acyclovir B.Erythromycin C. Mebendazole D. Permethrin/ prep. by Dr. Fekadu .T 8/27/2021 138

25-A 5 year-old boy sustained apenetrating injuryto his right leg . Two weeks later he develop swelling of the injured leg and high fever. X-ray of the affected limb showed features of acute osteomyelitis . What is the most likely etiology in the above case scenario ? A. pseudomonas aeurginosa B. Klebsiella pneumonia C. Streptococcus pyogenes/ D. E.coli prep. by Dr. Fekadu .T 8/27/2021 139

26-A 36 month-old male infant presented with high grade fever and vomiting of two days duration. He has right costovertebral angle tenderness. White cell count showed leukocytosis with left shift and blood film is negative forhemoparasites . What is the most likely diagnosis in this child? A. Pyelonephritis./ B. Hepatitis C. Appendicitis D. Lobar pneumonia prep. by Dr. Fekadu .T 8/27/2021 140

27-A 14 –month old male child is brought to the ERwith watery diarrhea of 24 hours duration. Physical examination reveals irritable infant, weight= 12 kg. sunken eye balls and eagerness to drink. What is the correct type and volume of fluid for this infant ? A. Ringer's lactate,300ml B.0.4% Normal saline, 700ml C. oral rehydration solution, 900ml/ D. Ringer's lactate, 500ml then ORS 500ml prep. by Dr. Fekadu .T 8/27/2021 141

28-which of the following vaccines should NOT be given in an infant with severe immune suppression ? A. Hepatitis B Vaccine B. Measles Vaccine/ C. Meningococcal vaccine prep. by Dr. Fekadu .T 8/27/2021 142

28-An 8- year-old boy known cardiac patient developed lowgrade fevwr , fatigue myalgia in the last two weeks.On examination he was found to have developed a new murmur, petechiael rashes and mild splenomegaly, which one is the most likely diagnosis? A. Acute rheumatic fever B. Dilated cardiomyopathy C. infective endocarditis/ D. Rheumatic heart disease prep. by Dr. Fekadu .T 8/27/2021 143

29-A 9 month old boy is brought to a health centerin a unresponsive state after repeated episodes of vomiting and diarrhea . Physical examination showed a rapide and feeble pulse , skin pich returns very slowely . Multiple attempts to scure an iv line have failed. What is the next best step in the management of this infant? A. Do blood film and determine blood glucose B. Give ORS through NG-tube and refer / C. obtain blood sample for complete blood count D. perform a lumbar puncture prep. by Dr. Fekadu .T 8/27/2021 144

30-A 6 year old diabetic child admitted to a Hospital with a diagnosis of severe diabetic acidosis has been receiving normal saline and insulin for the last three hours. Which electrolyte abnormality is expected? A. hypomagnesemia B. Hyponatremia C.Hypokalmia / D. HYpophosphatemia prep. by Dr. Fekadu .T 8/27/2021 145

Peripheral neuritis (manifesting as paresthesias of the hands and feet) which is the most common adverse effect of isoniazide might be due to deficiency of which vitamin? A-Ascorbic acid B-Niacin C-Pyridoxine D-Folic acid prep. by Dr. Fekadu .T 8/27/2021 146

Internal M –II MCQ prep. by Dr. Fekadu .T 8/27/2021 147

1-57 years old known hypertensive patient presented with exertional dyspnea of 2 weeks duration. On examination there is dullness on the Right lower 1/3 of posterior chest with decreased tactile fremitus and PMI shifted to lateral of left MCL. Which one is true regarding this patient? A.No need for pleural fluid analysis B.Normal CXR is expected in this patient C.ACE inhibitors do not have benefit for this patient D.The precordial examination findings are normal prep. by Dr. Fekadu .T 8/27/2021 148

Hypertension — The following definitions were suggested in 2003 by the seventh report of the Joint National Committee (JNC 7) based upon the average of two or more properly measured readings at each of two or more visits after an initial screen [ 4 ]: Normal blood pressure: systolic <120 mmHg and diastolic <80 mmHg Prehypertension : systolic 120 to 139 mmHg or diastolic 80 to 89 mmHg Hypertension: Stage 1: systolic 140 to 159 mmHg or diastolic 90 to 99 mmHg Stage 2: systolic ≥160 or diastolic ≥100 mmHg Read -HTN Risk fuctors c/m Ivx Treatment Complications prep. by Dr. Fekadu .T 8/27/2021 149

2-A thirty five year old man presents in a clinic with history of chronic productive cough that is worse in the morning and brought on by changes in posture. Sputum is copious and yellow . The most likely diagnosis in this patient is a? A)Bronchial asthma. B). Bronchiectasis . c) Bronchogenic carcinom D)Chronic bronchitis. e)Pulmonary tuberculosis. prep. by Dr. Fekadu .T 8/27/2021 150

Asthma Hx & PE: Younger patient, history of atopy , family Hx of asthma or atopy , episodic dyspnea , wheeze, cough, presence of specific trigger Dx : Spirometry COPD Hx & PE: Older patient (usually >50years). History of smoking or occupational exposure industrial fumes, biomass fuel, history of chronic dyspnea , cough, sputum production Dx : Spirometry Bronchiectasis Hx & PE: Hx of chronic cough productive of copious purulent sputum, repeated respiratory infection Ix: CXR and Chest CT prep. by Dr. Fekadu .T 8/27/2021 151

Pneumonia Hx & PE: Fever, pleuritic chest pain, productive cough, focal crackle on lung auscultation, other signs of consolidation Dx : CXR and Leukocytosis TB Hx & PE: High risk population (e.g., Immuno -compromised pt., patient living in endemic areas and contact Hx ), night sweat, weight loss, fever, cough Dx : CXR and sputum culture and AFB Lung cancer Hx & PE: Older patient, smoking history, weight loss, decreased appetite, chronic cough, hemoptysis , personal or family history of malignancies, paraneoplastic syndrome, hoarseness of voice prep. by Dr. Fekadu .T 8/27/2021 152

3-A twenty year old girl is taking anti-tuberculosis treatment. She presents in eye outdoor with visual complaints. The most likely cause of her symptoms is side effect of: a) Isoniazid . b) Rifampicin . c). Ethambutol . d) Pyrazinamide . e)Streptomycin . prep. by Dr. Fekadu .T 8/27/2021 153

prep. by Dr. Fekadu .T 8/27/2021 154

4-A forty year old lady gives history of weight gain and hoarseness of voice. On examination her pulse is 64 per minute and skin is pale, coarse and dry. The most important investigation to find diagnosis in this case is: a) Adrenocorticotrophin hormone (ACTH). b) Cortisol level. c) Gonadotrophin levels. d)Insulin like growth factor (IGF). e)Thyroid function test. prep. by Dr. Fekadu .T 8/27/2021 155

A history of prominent weight gain, cold intolerance, constipation, lack of concentration and lethargy suggest hypothyroidism Can cuse diastolic heart failure Clinical features c/f Symptoms  Patients could remain asymptomatic for several years or they might not recognize the symptoms themselves.  Intolerance to cold environments, constipation, weight gain, hair loss, dry skin  Hoarse voice, lethargy, memory loss, depressed reflexes, dementia  Abnormal menstrual periods and sub-fertility (in adult females) Signs  Puffy face, pallor, slow pulse (usually <60 per minute)  Goiter may be present prep. by Dr. Fekadu .T 8/27/2021 156

Ivx TSH is the best screening test o TSH >20 micro unit/ml : highly suggestive primary hypothyroidism o Normal TSH: excludes primary hypothyroidism o Mildly elevated TSH (<20microunits/ml): needs freeT4 determination o Mildly elevated TSH with normal free T4: subclinical hypothyroidism Investigations  TSH  Free T4 prep. by Dr. Fekadu .T 8/27/2021 157

5-A ten year old boy gives history of swelling of body starting from face and more on getting up in the morning. On examination his blood pressure is normal, pallor is absent and jugular venous pressure is not raised. Signs of ascites and bilateral pleural effusion are found. The first line of investigation in this case is: a)Blood urea level. b)Chest X-ray . c)Echocardiography . d)Liver function test. e).Urine for albumin. prep. by Dr. Fekadu .T 8/27/2021 158

Nephrotic syndrome (NS) in adults Brief description  Nephrotic syndrome (NS) is a clinical sate which results from heavy (massive) proteinuria .  The presence of nephrotic syndrome (NS) is a definitive indicator of glomerular pathology.  Nephrotic syndrome is defined if all the following three clinical and laboratory findings are fulfilled 1. Heavy proteinuria : defined as urine protein >3000mg(3gm) in a 24hr urine protein 2. Hypoalbuminemia : Defined as serum albumin <3g/dl 3. Edema(edematous state) prep. by Dr. Fekadu .T 8/27/2021 159

Hyperlipidemia , usually severe, is a common finding. Some experts include it as requirement for the diagnosis of NS..  Some patients might have heavy proteinuria (>3000mg in 24 hour urine) but do not have the other features of the NS. They are said to have nephrotic range proteinuria , but not the nephrotic syndrome prep. by Dr. Fekadu .T 8/27/2021 160

6-A young girl comes in the cardiology ward with history of breathlessness and palpitations for last one year. After auscultation of precordium cardiology Registrar makes diagnosis of mitral stenosis . The most important sign on which this diagnosis is based is: a)Ejection systolic murmur. b)Mid diastolic murmur. c)Mid systolic click. d)Pan systolic murmur. e)Third heart sound. prep. by Dr. Fekadu .T 8/27/2021 161

MR TR AS PS- systoilic HF Ms TS AR PR- Dias HF prep. by Dr. Fekadu .T 8/27/2021 162

7-A fifteen year old boy presents with history of fever and arthritis.There is past history of similar symptoms one year back . A diagnosis rheumatic fever is made. The treatment of choice for this patient’s symptom is: a)Aspirin . b) Diclofenac . c)Ibuprofen . d) Paracetamol . e)Morphine . prep. by Dr. Fekadu .T 8/27/2021 163

Rheumatic fever, acute Acute rheumatic fever is a condition, generally classified as connective tissue disease or collagen vascular disease. It follows group A beta-hemolytic streptococcal throat infection by a latency period of about 3 weeks. It is the most common cause of acquired heart disease in children in developing countries prep. by Dr. Fekadu .T 8/27/2021 164

Diagnostic criteria (revised Jones criteria) Evidence of recent streptococcal infection - Elevated ASO-titer or other streptococcal Ab titres or positive throat swab for group A beta- hemolyticus streptococcus PLUS Two major manifestations or one major and two minor manifestations MAJOR MANIFESTATIONS o Carditis (occurs in about 50-60% of patients. It affects all the layers of the heart ( endocardium , myocardium and pericardium). o Migratory Polyarthritis o Erythema marginatum o Subcutaneous nodules o Sydehnam‘s chorea MINOR MANIFESTATIONS - Arthralgia - Fever >380C - Acute phase reactants (increased ESR/CRP) - ECG: prolonged PR interval prep. by Dr. Fekadu .T 8/27/2021 165

NOTE : Diagnosis of acute rheumatic fever is established with: 2 major criteria or 1 major and 2 minor criteria and supporting evidence for antecedent streptococcal pharyngitis (mandatory).Strict adherence to Jones criteria is not needed under the following situations: Sydenham‘s chorea Indolent carditis   Rheumatic fever recurrence prep. by Dr. Fekadu .T 8/27/2021 166

Polyarthralgia or monoarthritis can be take as major criteria in moderate-high risk population (population stratum is based on incidence of RF in school age children or prevalence of RHD in all ages Diagnosis of recurrence: 2 major, or 1 major+ 2 minor, or 3 minor (in moderate-high risk population) plus evidence of preceding GAS infection prep. by Dr. Fekadu .T 8/27/2021 167

Investigations o Blood: Haemogram (raised ESR) or C-reactive protein (CRP) o Chest X-ray, ECG, Echocardiography o If it is available, Antistreptolysin O(ASO) titer , Culture from throat swab Treatment Objectives o Alleviate symptoms o Prevent complication prep. by Dr. Fekadu .T 8/27/2021 168

Treatment Objectives o Alleviate symptoms o Prevent complication Non-pharmacologic - Place on bed rest and monitor for evidence of carditis - Withhold anti-inflammatory treatment till full clinical pictures appear - Emotional support, especially crucial when Sydenham‘s chorea is present - Counseling of the child and family on the nature of the disease, long-term management, prognosis and prevention of recurrence prep. by Dr. Fekadu .T 8/27/2021 169

Pharmacologic To eradicate any streptococci:  Benzathine benzylpenicillin dose Child < 30 kg: 0.6 MU IM stat, Child > 30 kg: 1.2 MU IM stat  Alternative (considered when patient is hypersensitive to penicillin) Erythromycin 40mg/kg/24 orally divided into 2- 4 doses for 4 days OR   Azithromycin 500mg orally on the first day, then 250mg daily for 4 days. To treat the inflammation Anti-rheumatic treatment Migratory polyarthritis and carditis without cardiomegaly : Aspirin 100mg/kg/24 hours divided into 4 doses, orally for 3-5 days, then 75mg/kg/24hours for 4 weeks. Carditis with cardiomegaly or congestive Heart Failure : prednisolone 2mg/kg/24hours divided into 4 doses orally for 2-3 weeks, and while tapering prednisolone , start Aspirin 75mg/kg/24 hours in 4 divided doses for 6 weeks. prep. by Dr. Fekadu .T 8/27/2021 170

If chorea:Valproate 10-20 mg/kg/day Secondary prophylaxis for RF has to be started for all patients with sydenham‘s chorea Phenobarbital 15-30mg po Q6-8 hrs Other alternatives: valproate , haloperidol, chlorpromazine, steroids Prophylaxis To prevent further Benzathine benzylpenicillin 1.2 MU IM every 4 weeks, Child <30 kg: 0.6 MU If allergic to penicillin:Erythromycin 10 mg/kg twice a day Duration of prophylaxis depends on severity of disease: Rheumatic fever without carditis prep. by Dr. Fekadu .T 8/27/2021 171

If allergic to penicillin:Erythromycin 10 mg/kg twice a day Duration of prophylaxis depends on severity of disease: Rheumatic fever without carditis : for 5 years or until age 18 or 21 years old. Carditis but no residual heart disease: for 10 years or until age 25 years old. Carditis with residual heart disease: untill age 40- 45 years or for life. Prevention o Early diagnosis and treatment of group A Streptococcus throat infection o Avoid overcrowding, good housing o Good nutrition prep. by Dr. Fekadu .T 8/27/2021 172

8-An old lady presents with history of fever and left sided chest pain for one month. Examination of respiratory system shows decreased chest movements, stony dull percussion note and absent breath sounds on left side. Her chest X-ray is likely to reveal : a)Collapse. b)Consolidation . c)Fibrosis . d)Pleural effusion. e) Pneumothorax . prep. by Dr. Fekadu .T 8/27/2021 173

Pleural Effusion d efinition: - pleural effusion is the presence/ accumulation of excess fluid in the pleural space. Normally 10– 20 ml of fluid is in the pleural space c lassified as transudates and exudates Transudative effusion: - Heart failure , Cirrhosis of the liver Nephrotic syndrome , Myxoedema , Hypoproteinemia Exudative effusion: - Parapneumonic effusions , Empyema , Pulmonary embolism , Neoplasms , SLE, Sub-diaphragmatic abscess prep. by Dr. Fekadu .T 8/27/2021 174

Pleural Effusion DX Light’s Criteria Pleural fluid protein/serum protein >0.5 Pleural fluid LDH/serum LDH >0.6 Pleural fluid LDH more than two-thirds normal upper limit for serum (115–221 U/L) prep. by Dr. Fekadu .T 8/27/2021 175

9-A forty year old woman gives history of fever for last three weeks accompanied by dry cough, night sweats and weight loss . Chest examination is normal. Abdominal examination reveals hepatosplenomegaly . Chest X ray shoW symmetrically distributed fine nodules. The most likely diagnosis is: a)Military tuberculosis. b)Chronic liver disease. c)Malaria . d)Pneumonia . e)Typhoid . prep. by Dr. Fekadu .T 8/27/2021 176

10-A young girl complains of nocturnal cough and shortness of breath which disturbs her sleep. A diagnosis of bronchial asthma is made. The most important investigation to confirm this diagnosis is: a)Chest X-ray . b) Eosinophil count. c).Lung function tests. d)Serum IgGevels . e)Sputum examination. prep. by Dr. Fekadu .T 8/27/2021 177

11-A fifteen year old girl presents with history of fever, bleeding From gums and pallor for last fifteen days. Her peripheral blood film shows pancytopenia . The most important investigation is: a).Bone marrow examination . b) Coomb’s test. c) Reticulocyte count. d)Serum folic acid level. e)Serum iron level. prep. by Dr. Fekadu .T 8/27/2021 178

12-Which of the following drugs is used in the treatment of hyperkalemia (5.0-5.5 mEq /l) in acute renal failure: a) Amiloride . b) Amlodipine . c) Captopril . d).Insulin . e) Propranolol . prep. by Dr. Fekadu .T 8/27/2021 179

12-For the patient with history of fever, headache and neck stiffness , the most important investigation is: a).Cerebrospinal fluid examination. b)Complete blood counts. c)Computerized tomography scan brain. d)Magnetic resonance imaging brain. e)X-ray skull. prep. by Dr. Fekadu .T 8/27/2021 180

14-The most common risk factor for chronic obstructive pulmonary disease (COPD)is : a)Air pollution. b)Coal mining. c)Infection . d)Low socioeconomic status. e).Tobacco smoke. prep. by Dr. Fekadu .T 8/27/2021 181

15-In a patient with centripetal obesity, acne and hirsuitism . The most likely diagnosis is: a).Cushing’s syndrome. b)Diabetes mellitus. c) Hypogonadism . d)Hypothyroidism . e)Simple obesity. prep. by Dr. Fekadu .T 8/27/2021 182

Cushing‘s syndrome is a clinical syndrome which results from high levels. of cortisol in the blood and is associated with various changes in the body. It results in unexplained and rapid weight gain resulting in the development of obesity,hypertension , diabetes and osteoporosis. The major causes are pituitary tumor/adenoma , adrenal tumor or prolonged and excessive intake corticosteroids. Clinical features/Symptoms Symptoms could be subtle as they develop very slowly or asymptomatic -Weight gain-Change in body habitus and shape: obesity, facial fullness -Excessive facial hair - Easy fatigability - Easy bruising of the skin prep. by Dr. Fekadu .T 8/27/2021 183

Diagnosis The diagnosis of Cushing‘s syndrome starts with high index of clinical suspicion;however , confirmation requires biochemical tests. -The first step in the diagnosis is to exclude exogenous steroid use (oral, IM, IV) -Whom to screen with biochemical tests? -Those with clinical features highly predictive of Cushing‘s syndrome -Osteoporosis or hypertension in young adults -Incidental adrenal mass The biochemical testing requires the following three steps 1. Confirming high cortisol 2. Determine if the high cortisol is ACTH-dependent or ACTH-independent 3. Determine the source of ACTH ( if it is ACTH-dependent) Patients with suspected Cushing‘s syndrome should be referred either directly or after doing screening test prep. by Dr. Fekadu .T 8/27/2021 184

Screening biochemical test options 1. 1mg overnight dexamethasone suppression test - Give 1 mg of dexamethasone at 11 PM to 12 AM (midnight), and measurement of serum cortisol at 8 AM the next morning. - A morning cortisol level above 1.8 mcg/dl (50nmol/l) is suggestive of Cushing‘s syndrome. 2. 24 hour urine free cortisol : Level 3-4 times the upper limit of normal is highly suggestive 3. Late evening(11 PM) salivary cortisol prep. by Dr. Fekadu .T 8/27/2021 185

Treatment Objectives of treatment Normalize plasma level of cortisol Correct metabolic and electrolyte abnormalities Correct blood pressure Non pharmacologic treatment Surgery: the main stay of treatment of Cushing‘s syndrome is surgery (pituitary or adrenal gland surgery depending on the cause) The surgery should be done in referral hospitals with experience in doing surgery. Pharmacologic Manage hypertension and diabetes as per the standard treatment guidelines and refer patient for definitive treatment prep. by Dr. Fekadu .T 8/27/2021 186

investigations - Screening biochemical test options  1. 1mg overnight dexamethasone suppression test - Give 1 mg of dexamethasone at 11 PM to 12 AM (midnight), and measurement of serum cortisol at 8 AM the next morning. - A morning cortisol level above 1.8 mcg/dl (50nmol/l) is suggestive of Cushing‘s syndrome. 2. 24 hour urine free cortisol : Level 3-4 times the upper limit of normal is highly suggestive 3. Late evening(11 PM) salivary cortisol prep. by Dr. Fekadu .T 8/27/2021 187

16-In a young boy with hypertension, examination of cardiovascular system reveals radio-femoral delay. The most Likely cause of hypertension in this patient is: a). Coarctation of aorta. b)Diabetic nephropathy. c)Conn’s syndrome. d)Dissection of aorta. e)Renal artery stenosis . prep. by Dr. Fekadu .T 8/27/2021 188

Congenital heart disease Congenital heart disease is the functional or structural defect that is present at birth. The cause is usually unknown but has been associated with several teratogen factors such as maternal infections (rubella) maternal medications (folic acid antagonists, estrogens, anticonvulsant) and maternal diabetes. - Congenital heart disease can be divided as cyanotic and acynanotic . A)The mostcommon cyanotic heart diesases are (5 TS) T etratology of Fallot (TOF), T ransposition of great arteries, T ricuspic / pulmonary atresia , T runcusarteriosus , T otal anomalous pulmonary venous drainage ( 5T’s ). B)The mostcommon acyanotic heart diseases are ventricular septal defect, patent ductus arteriosus , atrial septal defect, -pulmonary / aortic stenosis and coarcation of aorta. prep. by Dr. Fekadu .T 8/27/2021 189

Common causes heart failure in children . Left to right shunt and regurgitant lesions . Primary myocardial dysfunction such as myocarditis , cardiomyopathy . Ductal dependent lesions such as Hypoplastic left heart syndrome, severe coarctation of the aorta or interruption of the aortic arch. . Single ventricle with no stenosis prep. by Dr. Fekadu .T 8/27/2021 190

RISK Factors I-Prenatal and natal history Maternal health during pregnancy ▪ Fever and infectious diseases • Rubella in 1st trimester causes congenital heart disease • Coxsackie B in late trimester may cause myocarditis in the fetus • HIV is associated with cardiomyopathy II- Family History - Congenital heart disease in parents Chance can increase up to 15% - Maternal diabetes Increases chance of cardiomyopathy prep. by Dr. Fekadu .T 8/27/2021 191

III-Medication and drugs during pregnancy ▪ Anticonvulsants • Phenytoin (pulmonary stenosis , aortic stenosis , coarctaion of aorta and PDA) • Valproic acid ( atrial and ventricular septal defect, aortic stenosis , pulmonary atresia with intact ventricular septum and coarctaion of the aorta. • Lithium (Epstein anomaly) • Methadone: TGA, TOF and hypoplastic left heart syndrome • Amphetamines: ventricular and atrial septal defect, PDA, TGA • Alcohol and smoking: fetal alcohol syndromes associated with atrial septal defect (ASD), ventricular septal defect and TOF o Labor and delivery Duration of labor ▪ Type of delivery ▪ Problems after delivery • Did the baby cry immediately? • Was the baby breathing well? • Was the baby put in the NICU o MAS: results in right to left shunting through patent foramen ovale Severe birth asphyxia may cause myocardial ischemia and failure Down syndrome is a common syndrome associated with congenital heart disease in our setup prep. by Dr. Fekadu .T 8/27/2021 192

Coarctation of Aorta C/F Cough .SOB . Headache.Feeding difficulty -High blood pressur P/E Weak leg pulses and strong arm pules ( coarctation of aorta) Radio -Femoral Delay BP discrepancy b/n upper and lower extremities reduce or absent pulse in the lower extrimity Hypertension is late presentation so compare Bp in all four extrimities Best DX in infants adolescents and adults clinically by palpating simultaneousl femoral and brachieal pulses -IVX CXR - Dilated descending aorta - Enlarged LV - Rib notching prep. by Dr. Fekadu .T 8/27/2021 193

ECG - Left ventricular hypertrophy (can be normal in childhood) ECHO- Measurement of stenotic area - Pressure gradient Mangt - Coarcation of Aorta - PGE1 infusion to maintain patent duct (in neonate) - Surgery after stabilization or diagnosis of patient - If diagnosed in adult, treat hypertension and heart failure and follow with surgery prep. by Dr. Fekadu .T 8/27/2021 194

17-The gait of a patient with cog-wheel rigidity and pill rolling tremors is likely to be: a)Drunken . b)Hemiplegic . c)High stepping. d)Normal E)Shuffling. prep. by Dr. Fekadu .T 8/27/2021 195

20-An old patient presented in emergency ward with history of weakness of right side of body of rapid onset. The most helpful first line investigation for management of this patient is: a)Cerebral angiography. b) Cerebro spinal fluid examination. c)Computerized tomography scan brain. d)Fasting lipid profile. e)Nerve conduction study. prep. by Dr. Fekadu .T 8/27/2021 196

21-In a patient of thalessemia peripheral blood film for red cell morphology shows: a). Hypochromic microcytic cells. b) Normochromic normocytic cells . c) Macrocytosis . d)Sickle cells. e) Spherocytes . prep. by Dr. Fekadu .T 8/27/2021 197

22-An epileptic girl is found to have gum hypertrophy. Anti-epileptic drug which she is most likely taking is: a) Carbamazepine.b b) Gabapentin . c) Lamotrigine . d). Phenytoin . e)Sodium valproate . prep. by Dr. Fekadu .T 8/27/2021 198

23-A patient of chronic diarrhea is having angular stomatitis and glossitis . The most likely cause of these signs is deficiency of: a)Folic acid. b).Iron . c)Proteins . d)Thiamine . e)Pyridoxine . prep. by Dr. Fekadu .T 8/27/2021 199

24-In a patient with high grade fever, rigors and tender hepatomegaly . The most likely diagnosis is: a)Carcinoma of liver. b)Liver abscess. c)Malaria . d)Right heart failure. e)Typhoid fever. prep. by Dr. Fekadu .T 8/27/2021 200

25-A forty year old man gives history of high grade fever for last one week associated with cough productive of rusty sputum. Auscultation reveals bronchial breathing on right lower chest. Chest X-ray shows consolidation. The most likely causative organism is: a)Anaerobic bacteria. b)Gram negative bacilli. c)Mycobacterium tuberculosis. d)Staphylococcus aureus . e).Streptococcus pneumoniae . prep. by Dr. Fekadu .T 8/27/2021 201

26-The anti diabetic agent of choice for a fifty year old obese lady with mild hyperglycemia is: a) Chlorpropamide . b) Glibenclamide . c)Insulin . d) Metformin . e) Repaglinide . prep. by Dr. Fekadu .T 8/27/2021 202

27-A fifty year old man presents with dysphagia . Which of the following characteristic suggests a benign stricture of esophagus : a) Anaemia . b)Cervical lymphadenopathy . c) Dysphagia worse for solids. d)Hoarseness of voice. e)Weight loss. prep. by Dr. Fekadu .T 8/27/2021 203

28-A thirty five year old man presents with history of low grade fever and cough for last three months. Examination of respiratory system is normal. A diagnosis of tuberculosis is made . Which of the following feature on chest X-ray suggests this diagnosis : a) Cavitation . b)Consolidation. c) Hilar congestion. d)Prominent bronchovascular marking. prep. by Dr. Fekadu .T 8/27/2021 204

29-In a patient with history of shortness of breath,which of the following sign indicates left heart failure: a) Ascites . b)Basal crepitations . c)Dependant edema. d)Engorged neck veins. e)Fourth heart sound. prep. by Dr. Fekadu .T 8/27/2021 205

30-A fifty year old smoker presents with history of cough productive of mucoid sputum in every winter for last three years.The most likely diagnosis is: a)Bronchial asthma. b) Bronchiectasis . c) Bronchogenic carcinoma. d)Chronic bronchitis. e)Pulmonary tuberculosis. prep. by Dr. Fekadu .T 8/27/2021 206

31-A forty year old lady presents with history of severe, constant upper abdominal pain that does not radiate and is associated with vomiting. On examination temperature is normal and there is marked tenderness in epigastrium . Most useful investigation for this patient is: a)Cardiac enzymes. b)Electrocardiography . c) Gastroscopy . d)Liver function tests. e)Serum amylase. prep. by Dr. Fekadu .T 8/27/2021 207

32-In a patient with history of haematemesis , the clinical feature which suggests that peptic ulcer is : a) Ascites . b)Drowsiness . c)History of jaundice. d)Tender epigastrium . e) Splenomegaly . prep. by Dr. Fekadu .T 8/27/2021 208

33-A young boy presents with history of fever, skin rash and diarrhea . Examination of oral cavity shows koplik spots on buccal mucosa. The most likely diagnosis is: a)Chicken pox. b)Diphtheria . c)Measles . d)Small pox. e)Typhoid . prep. by Dr. Fekadu .T 8/27/2021 209

34-A patient presents with history of intermittent fever, abdominal pain and headache. Abdominal examination shows tenderness and hepatosplenomegaly . His blood culture is positive for salmonella typhi . The antibiotic of choice for this patient is: a)Ciprofloxacin . b) Gentamycin . c) Metronidazole . d)Tetracycline . e) Vancomycin . prep. by Dr. Fekadu .T 8/27/2021 210

36-A 60 year old male patient presented to you with chief complaint of loss of consciousness of four hrs Duration . The attendants gave you history of headache, neck stiffness and high grade fever of five day duration. He has history of abnormal body movement involving all extremities of one episode staying five minutes . His vital sign: Blood Pressure: 140/90 mmHg .Pulse Rate: 56 RRate : 34 irregular T: 38.4 .Pupils are 7 mm bilaterally in size and reactive . Which of the following is false about this patient? A.This patient is likely to have increased intracranial pressure. B.Has poor prognosis features. C.Listeria Monocytogenes is likely to be considered as one of etiologic agent. D.Antibiotics shouldn’t be started blindly unless gram stain is done because this will promote drug resistance. prep. by Dr. Fekadu .T 8/27/2021 211

37-A 62-year-old woman is admitted to the hospital with a community-acquired pneumonia with a 4-day history of fever, cough, and right-sided pleuritic chest pain. The admission chest x-ray identifies a right lower and middle lobe infiltrate with an associated effusion. All of the following characteristics of the pleural effusion indicate a complicated effusion that may require tube thoracostomy EXCEPT: A= Loculated fluid B-Pleural fluid pH < 7.20 C-Pleural fluid glucose <60 mg/ dL D-Positive Gram stain or culture of the pleural fluid E-Recurrence of fluid following the initial thoracentesis prep. by Dr. Fekadu .T 8/27/2021 212

prep. by Dr. Fekadu .T 8/27/2021 213

38-All of the following are minor criteria in the Modified Duke Criteria for the clinical diagnosis of infective endocarditis EXCEPT: A-Immunologic phenomena ( glomeluronephritis , Osler nodes, Roth spots) B-New valvular regurgitation on transthoracic echocardiogram C-Predisposing condition (heart condition, intravenous drug use) D-Temperature >38°C E-Vascular phenomena (e.g., arterial emboli, septic pulmonary emboli, Janeway lesions) prep. by Dr. Fekadu .T 8/27/2021 214

Infective endocarditis (IE) is an infection of the endocardium and/or heart valves that involves thrombus formation (vegetation), which may damage the endocardial tissue and/or valves. History of predisposing factors: o Fever, Sore throat, Dental procedures, Underlying congenital or acquired valvular heart disease o Recent cardiac surgery, IV drug misuse , Skin infection ,Central venous catheter or prosthetic valves - Immunocompromised state (use of immunosuppressive medications, chemotherapy or co-existing diseases such as AIDS). prep. by Dr. Fekadu .T 8/27/2021 215

- C/M Dyspnea , palpitation, cough, chest discomfort due to the underlying heart disease, i.e. valvular or congenital lesion -Fever with chills, nausea, anorexia, hemoptysis , --headache, pallor, lassitude, fatigue, -weakness, diaphoresis due to systemic inflammatory response -Onset and duration of symptoms prep. by Dr. Fekadu .T 8/27/2021 216

PHYSICAL EXAMINATION General: The patient looks toxic, wasted. Vital signs: Febrile or hypothermic in rare cases o Pulse rate can be elevated, o Usually normotensive but can be in shock if there is sepsis Respiratory rate is usually elevated. H.E.E.N.T : Eyes : Sub- conjunctival hemorrhage, Roth’s spots, conjunctival pallor, redness of the eye,icteric sclera Throat: Evidence of sore throat Mouth: Dental caries or bleeding gum. Lymphoglandular system : Lymphadenopathy Respiratory system: General: Clubbing and cyanosis Chest examinations: • Pleural friction rub due to embolic pulmonary infarct may be present • Bi-basilar rales because of heart failure • Signs suggestive of pneumonia and consolidation because of septic embolization CVS: Arterial : Loss of peripheral pulses due to embolization Venous: Raised JVP Precordial : Changing or appearance of new murmurs, signs suggestive of CHF, muffling of heart sounds GI system : Splenomegaly , hepatomegaly , splenic or hepatic rub because of infarction GU system: CVA tenderness Integumentary system: o Fingers and nail: Digital gangrene, clubbing of the fingers, splinter haemorrhages , Osler’s nodes (painful tender swellings at fingertips) o Janeway’s lesion (large nontender maculopapular eruptions in palm and sole). o The skin may show purpuric spot and ecchymosis . Musculoskeletal system: Pedal edema and cold extremities. CNS : Monoplegia , hemiplegia or other focal neurologic deficits because of embolization . prep. by Dr. Fekadu .T 8/27/2021 217

prep. by Dr. Fekadu .T 8/27/2021 218

39-Which one of the following is the first step in approaching a child presenting with seizure? A.Search for potentially life-threatening causes of seizure and treating them B.Evaluating the patient to determine whether the seizure has a focal onset of is generalized C.Managing the patient according to the ABC approach D.Giving IV diazepam prep. by Dr. Fekadu .T 8/27/2021 219

40-Most common route of transmission of UTI in children is A.Sexual abuse B.Ascending infection C.Hematogenous spread D.None prep. by Dr. Fekadu .T 8/27/2021 220

41-For a patient having typical clinical manifestation of congestive heart failure, what investigation can we send to support our diagnosis? A.CXR B.Echocardiography C.ECG D.All of the above prep. by Dr. Fekadu .T 8/27/2021 221

42-Which one the following is not common complication of malaria in pregnancy A. High- level parasitemia with anemia B. Hypoglycemia C. Acute pulmonary edema. D.. Convulsions prep. by Dr. Fekadu .T 8/27/2021 222

43-one of t he following management in tetanus used for eradication of c.Tetani ? A. Human tetanus immune globulin (t Ig ) B. Tetanus antitoxin C. Metronidazole D. Diazepam prep. by Dr. Fekadu .T 8/27/2021 223

Peripheral neuritis (manifesting as paresthesias of the hands and feet) which is the most common adverse effect of isoniazide might be due to deficiency of which vitamin? A-Ascorbic acid B-Niacin C-Pyridoxine D-Folic acid prep. by Dr. Fekadu .T 8/27/2021 224

PEDIATRICS –PART II MCQ prep. by Dr. Fekadu .T 8/27/2021 225

1-A 2- year old child has delay in developmental milestones for his age. On physical examination he has macrocephaly , severe stunting, protruded tongue, umbilical hernia and coarse facial features. Which investigation is very helpful in managing this patient? A. Cortisol B. Growth hormone C.TSH D. vitamin D prep. by Dr. Fekadu .T 8/27/2021 226

2-which of the following best describe clinical features of Guillain-Barre syndrome? A, Ascending symmetric paralysis withoutsensory change B. Asymmetric paralysis and sensory change C. Symmetric paralysis and centeral nerve palsy D. symmetric paralysis and sensory change prep. by Dr. Fekadu .T 8/27/2021 227

Hypothyroidism - Hypothyroidism may occur at birth (congenital hypothyroidism) or at any time during childhood or adolescent (juvenile hypothyroidism). - Congenital hypothyroidism - Deficiency of thyroid hormone present at or before birth. Etiology - Primary: Agenesis or dysgenesis , ectopic, dyshormonogenesis - Secondary : Hypopituitarism , hypothalamic abnormality - Other: Transient, maternal factors (e.g., goitrogen ingestion, iodide deficiency) Identification - Age - Congenital Hypothyroidism is the most common neonatal endocrine disorder - Sex - Twice as common in girls as in boys - Address - regions of iodine deficiency History Symptoms of possible congenital hypothyroidism include: - Lethargy, sleepiness, - Poor feeding, - Constipation, chronic? - Hoarse cry – due to myxomatous infiltration of vocal cords - Mottling, cold extremity - Cough, fever, noisy breathing, breathlessness – due to repeated LRTI - Swelling over the neck – goiter - Swelling over the body – myxedema - History of risk factors should also be included such as o History of drugs: antithyroid drugs or iodine intake during pregnancy. o Surgery / irradiation o History of living area for endemic goiter prep. by Dr. Fekadu .T 8/27/2021 228

Family History - Similarly affected infant - Family members with unexplained mental retardation Developmental History - Any history of developmental delay (when were all the major milestones achieved?) – detailed Clinical presentation -Most appear clinically normal at birth - maternal T4 provides fetal levels that are 33% of normal at birth -Clinical picture may be nonspecific, making clinical detection difficult and can become fully developed by 3-6 month of age On Physical Examination General Appearance: - Coarse facial features - Presence of yellowish discoloration, prep. by Dr. Fekadu .T 8/27/2021 229

On Physical Examination General Appearance: - Coarse facial features - Presence of yellowish discoloration, Vital Signs : - Bradycardia 143 - Hypotension (Decreased systolic blood pressure and increased diastolic blood pressure) - Hypothermia H.E.E.N.T : - Macroglossia - Enlarged posterior fontanel (>1 cm) - Wide sutures - Coarse, brittle, straw like hair Lymphoglandular System - Goiter Cardiovascular System - A small but significant number of infants with congenital hypothyroidism have other birth defects, mainly atrial and ventricular septal defects. Look for signs of Sinus bradycardia , Pericardial effusion, Congestive cardiac failure, Cardiomyopathy . Abdominal - Umbilical hernia - Distended abdomen Integumentary system: - Mottled & dry skin, p prep. by Dr. Fekadu .T 8/27/2021 230

Myxedema (in the skin of the eyelids, the back of the hands, & the external genitals) Nervous system : - Hypotonia - Slow reflexe investigations -Lab investigations: T3, T4, TSH Clinical hypothyroidism: lowT4, T3, and high TSH Subclinical hypothyroidism: Normal T4, T3 and High TSH -Thyroid scan -Thyroid ultrasound/ scintinigraphy -ECG: low-voltage P and T waves with diminished amplitude of QRS complexes suggest poor LV function and pericardial effusion -EEG: often shows low voltage -Elevated serum cholesterol level (in children >2 yr) prep. by Dr. Fekadu .T 8/27/2021 231

3-What is the compression to ventilation ratio in resuscitating a newborn? A. 3:1 B. 5:1 C. 15:1 D. 30:1 prep. by Dr. Fekadu .T 8/27/2021 232

Paediatric resuscitation is different to adult resuscitation in many aspects, but the basic principles remain the same: • Don’t panic • Ensure both the patient and rescuer are safe • Get help • Initiate basic life support at the earliest possible moment: Airway, Breathing, Circulation. DRSABC is a mnemonic to aid the sequence of events that should be followed when meet with any collapsed patient. Check for Danger to self and patient, check for patient Responsiveness, S end for help, open the A irway, check for B reathing, and if patient is not breathing normally give two rescue breaths; assess C irculation by checking for signs of life and if absent commence Cardiopulmonary resuscitation ( C PR). prep. by Dr. Fekadu .T 8/27/2021 233

Quality chest compressions involve compressing the lower half of the sternum to at least one-third of the anteroposterior chest diameter at a rate of 100–120 compressions per minute with subsequent complete release to allow recoil of the chest wall. 4 Basic life support rescuers and lone rescuers should use a 30:2 compressions to ventilation ratio, while advanced life support rescuers performing two-rescuer CPR should use a 15:2compressions to ventilation ratio for all children, except in the newly born infant where it is 3:1. prep. by Dr. Fekadu .T 8/27/2021 234

4-which of the following is the correct percentage of dextroseto treat neonatal hypoglycemia? A. 5% B. 10%. C. 40% D. 50% prep. by Dr. Fekadu .T 8/27/2021 235

• Don’t panic • Ensure both the patient and rescuer are safe • Get help • Initiate basic life support at the earliest possible moment: Airway, Breathing, Circulation. 2 DRSABC is a mnemonic to aid the sequence of events that should be followed when meet with any collapsed patient. Check for Danger to self and patient, check for patient Responsiveness, S end for help, open the A irway, check for B reathing, and if patient is not breathing normally give two rescue breaths; assess C irculation by checking for signs of life and if absent commence Cardiopulmonary resuscitation ( C PR). prep. by Dr. Fekadu .T 8/27/2021 236

Metabolic Disorders of the Newborn  Hypoglycemia  Hyperglycemia  Thermoregulation  Hypocalcemia – discussed in fluid and electrolyte section prep. by Dr. Fekadu .T 8/27/2021 237

Correction of Metabolic States - - Blood glucose has to be kept in the normal range. Hypoglycemia is often seen in asphyxiated newborns. It increases energy deficit. It has to be treated with 2ml/kg of 10% dextrose 4ml/kg (in the presence of seizure) followed by maintenance. - Hypocalcemia (can cause seizure and decreased cardiac contractility) administer 1- 2ml/kg of 10% calcium gluconate QI prep. by Dr. Fekadu .T 8/27/2021 238

5-which of the following CSF profile is expected in a child with meningoencephalitis ? A.Leukocytosis with neutrophil predominance B. Normal protein level C. normal or slightly reduced glucose level D. organism on Gram stain prep. by Dr. Fekadu .T 8/27/2021 239

Hypoglycemia Hypoglycemia is common metabolic problem in NICUs. This is because of abrupt cease in glucose supply following clamping of the umbilical cord at birth . This variability is due to number of factors including:  Gestational age Birth weight  Post natal age  Feeding status  Presence or absence of associated illnesses The diagnosi hypoglycemia could be defined as blood glucose level less than 40mg/dl. There are two types of neonatal hypoglycemia, transient and persistent. Most neonates will have transient hypoglycemia, which responds to treatment and is associated with good prognosis. Causes of hypoglycemia Transient hypoglycemia could be: I. Related with changes in maternal metabolism Intrapartum glucose administration Diabetes in pregnancy-infant of diabetic mother  Maternal drugs ( tocolytics , propranolol , thiazide diuretics) II. Related with neonatal problems Intrauterine growth retardation Prematurity Delayed onset of feeding Birth asphyxia Infection Post exchange transfusion  Hypothermia  Delayed feeding Polycythemia  Erythroblastosis fetalis prep. by Dr. Fekadu .T 8/27/2021 240

6-A 5 year –old boy with HIV infection is receiving Zidovudine , Lamivudine and Nevirapine . He presents with progressive difficulty of breathing for three weeks. He has no history of fever. on examination PR=108/min RR=72/min T=37oc and oxygen saturation is 70%.chest radiograph reveals bilateral diffuse alveolar disease. What is the most likely diagnosis? A. Heart failure B). pneumocystis pneumonia C. pulmonary tuberculosis D. severe bacterial pneumonia prep. by Dr. Fekadu .T 8/27/2021 241

Management of neonatal hypoglycemia The overall management of neonatal hypoglycemia should include: 1. Anticipation and prevention in those who are at high risk. 2. Correction of hypoglycemia in those who are symptomatic and 3. Investigation and treatment of the cause of hypoglycemia, when it is possible to identify the cause. a) Treatment of asymptomatic hypoglycemia Feeding Feeding is the initial treatment in an asymptomatic term infants, - Immediately offer breast-feeding. - Check blood glucose30 minutes after feeding to insure normal glucose level before the next feeding. - If repeated blood glucose is > 40mg/dl continue to offer feedings at 2-3hours interval. Indications of IV infusions in asymptomatic hypoglycemia (use same infusion as symptomatic hypoglycemia)  Blood glucose < 25mg/dl.  Blood glucose remains < 40mg/dl after one attempt of feeding  If infant becomes symptomatic  If oral feeding prep. by Dr. Fekadu .T 8/27/2021 242

b) Treatment of symptomatic hypoglycemia Many neonates have asymptomatic (chemical) hypoglycemia. In contrast to the frequency of chemical hypoglycemia, the incidence of symptomatic hypoglycemia is highest in small for gestational age infants. The exact incidence of symptomatic hypoglycemia has been difficult to establish because many of the symptoms in neonates occur together with other conditions Immediate treatmen t Secure IV line  Give 2ml/kg of 10% glucose IV bolus over one minute if signs other than seizure Give 4 ml/kg of 10% glucose as a bolus over one minute if seizure is present. The small bolus minimizes hyperglycemia that can provoke insulin secretion and possibly prolong hypoglycemia. 10% dextrose for IV bolus can be prepared using 40% dextrose, which is available in our country by taking one part of 40% dextrose and three parts of distilled water prep. by Dr. Fekadu .T 8/27/2021 243

7-A 3 month old infant has rhinorrhea caugh and noisy breath. Physical examination reveals RR=72/min PR=142/min and T=37.80c he is irritable and has diffuse wheezes and chest retraction. What is the most likely diagnosis? A. Bronchiolitis B. Gastro-esophageal reflux disease C.Pertusis D. severe pneumonia prep. by Dr. Fekadu .T 8/27/2021 244

SCAP Cough or diffi culty in breathing with: • Oxygen saturation < 90% or central cyanosis • Severe respiratory distress e.g. grunting, severe chest indrawing • Signs of pneumonia with • a general danger sign (inability to breastfeed or drink, lethargy or reduced level of consciousness, convulsions) Pnemonia Fast breathing ≥ 60 breaths/minute in a Child aged <2 months; ≥ 50 breaths/ minute in a child aged 2–11 months ≥ 40 breaths/minute in a child aged 1–5 years OR • Chest indrawing • Definite chest findings on auscultation Bronchiolitis , a lower respiratory tract infection (LRTI) that primarily affects the small airways (bronchioles). prep. by Dr. Fekadu .T 8/27/2021 245

C/F The illness begin with URT symptoms and progress rapidly over 1-2 days to the development of increased respiratory effort and wheezing . Signs and symptoms includes: Tachypnea , Cough, Irritability, Fever Poor feeding, Nasal flaring Episodes of apnea (more common in infants). Retractions ( intercostal , subcostal , sternal ). DX; The diagnosis of acute bronchiolitis is clinical. Chest radiography can reveal hyperinflated lungs with patchy atelectasis . The white blood cell and differential counts are usually normal. prep. by Dr. Fekadu .T 8/27/2021 246

8-A 2- year old child brought to you with history of labored breathing associated with barking cough and inspiratory stridor . What will be the most likely problem of the child? A. Pneumonia B. Croup C. Very severe disease D. Asthma prep. by Dr. Fekadu .T 8/27/2021 247

The term croup refers mainly acute and infectious processes that are characterized by a bark-like or brassy cough and may be associated with hoarseness, inspiratory stridor , and respiratory distress. Croup also called laryngotracheobronchitis . Croup typically affects the larynx, trachea, and bronchi. Most common cause of infectious acute upper airway obstruction. Most patients with croup are between ages of 3 mo and 5  yr, with the peak in the 2nd yr of life. Prevalence-affects 3-5% of children. M:F is 2:1. Occurs commonly in the late fall and winter but can occur throughout the year. Recurrences are frequent from 3-6 yr of age . prep. by Dr. Fekadu .T 8/27/2021 248

ETIOLOGY The parainfluenza viruses (types 1, 2, & 3) account for ∼75% of cases. Other viruses includes influenza A and B, adenovirus, respiratory syncytial virus (RSV), and measles. DX Preceding URIs symptoms such as rhinorrhea , pharyngitis , mild cough, and low-grade fever for 1-3 days. Then the child develops the characteristic “barking” cough, hoarseness, and inspiratory stridor . Symptoms are characteristically worse at night and often recur , Agitation and crying greatly aggravate the symptoms and signs. prep. by Dr. Fekadu .T 8/27/2021 249

9-What is the recommended time to begin complementary feeding to infants? A. At 4 to 5 months of age B. At 6 months of age C. When the birth weight has tripled D. When tooth eruption has started prep. by Dr. Fekadu .T 8/27/2021 250

10-Which of the following is not the feature of respiratory distress syndrome (RDS)? A-Bluish discolouration of the mucous membrane B-Chest in drawing C-Flaring of the ala nasi D- Hyperventilation prep. by Dr. Fekadu .T 8/27/2021 251

11-Betty” who is 16 month of age came to your clinic with vomiting everything, unable to drink and suck breast, and has history of convulsion. During P/E you have found that she has severe chest- indrawing , stridor and breathing rate is 53 breaths per minute. What is the possible classification of Betty? A-Possible severe bacterial infection B-Pneumonia C-No pneumonia: cough or cold D-Severe pneumonia prep. by Dr. Fekadu .T 8/27/2021 252

12-Which one is the first line drug for an infant presented with pneumonia? A. Amoxicillin B. Ciprofloxacillin C. Cotrimoxazole D. Ceftriaxone prep. by Dr. Fekadu .T 8/27/2021 253

13-Which of the following fluid is preferred for a child presented with both dehydration and malnutrition with edema? A. Ringer lactate B. ORS C. ReSoMal D. Normal saline prep. by Dr. Fekadu .T 8/27/2021 254

14-Firew, aged 2 years is brought with diarrhea , fever and vomiting. On P/E, he is alert but irritable and restless with T =39 o C , eye-balls are not sunken and his skin pinch goes backquickly . Which of the following statement is correct? A. He has some dehydration B. He should be given antibiotics C. He should be treated according to treatment plan “B” D. He has no dehydration prep. by Dr. Fekadu .T 8/27/2021 255

15-Which of the following are the recommended immunizations for an infant coming in the second visit? A. Polio 2, penata 2, PCV2, Rota 2 B. OPV 1, penata 1, PCV1, Rota 1 C. BCG, Polio 2, penata 2, PCV2 D. Polio 2, penata 2, PCV2, Rota 1 prep. by Dr. Fekadu .T 8/27/2021 256

prep. by Dr. Fekadu .T 8/27/2021 257

16-What would be the most likely diagnosis of a child presented fever, macculopapular rash and conjunctivitis? A. Rubella B. Measles C. Scabies D. Chiken pox prep. by Dr. Fekadu .T 8/27/2021 258

17-According to WHO clinical staging HIV/ADS for infant and children, a child with esophageal candidiasis (or candida of the trachea) and recurrent severe bacterial infections could be classified as A. Stage I B. Stage II C. Stage III D. Stage IV prep. by Dr. Fekadu .T 8/27/2021 259

Case Definitions Measles - The following case definition for measles has been approved by the Council of State and Territorial Epidemiologists (CSTE) and was published in 1997 and updated in 2009. Clinical case definition - Measles is an illness characterized by all of the following: • A generalized maculopapular rash lasting ≥ 3 days • A temperature ≥ 101oF (38.3oC) • Cough, coryza , or conjunctivitis Laboratory criteria for diagnosis • Positive serologic test for measles immunoglobulin M ( IgM ) antibody; • Significant (generally a four-fold) rise in measles IgG antibody level by any standard serologic assay; • Isolation of measles virus from a clinical specimen; • Detection of measles –virus specific nucleic acid by polymerase chain reaction Note: Genotype identification by a WHO reference laboratory (CDC) is required to distinguish wild type from vaccine strains if the case was vaccinated within 18 days of rash onset. Case classification Suspected: Febrile illness accompanied by generalized, maculopapular rash. Probable: A case that meets the clinical case definition, has noncontributory or no serologic or virologic testing, and is not epidemiologically linked to a confirmed case. Confirmed: A case that is laboratory confirmed or that meets the clinical case definition and is epidemiologically linked to a confirmed case. A laboratory-confirmed case does not need to meet the clinical case definition. prep. by Dr. Fekadu .T 8/27/2021 260

Rubella - The case definition for rubella has been approved by the CSTE and was published in 2009. Clinical case definition - Rubella is an illness that has all of the following characteristics: • Acute onset of generalized maculopapular rash • Temperature >99°F (37.2°C), if measured • Arthralgia , arthritis, lymphadenopathy , or conjunctivitis Laboratory criteria for diagnosis • Isolation of rubella virus; or • Significant rise between acute- and convalescent-phase titers in serum rubella immunoglobulin G antibody level by any standard serologic assay; or • Positive serologic test for IgM antibody; or • Detection of rubella-virus specific nucleic acid by polymerase chain reaction. CDC Report: Elimination of Measles, Rubella, and CRS Case classification Suspected: Any generalized rash illness of acute onset that does not meet the criteria for probable or confirmed rubella or any other illness Probable: A case that meets the clinical case definition, has no or noncontributory serologic or virologic testing, and is not epidemiologically linked to a laboratory-confirmed case of rubella Confirmed: A case that is laboratory confirmed (with or without symptoms) or that meets the clinical case definition and is epidemiologically linked to a laboratory-confirmed case of rubella Importation status The same importation status classifications are used for measles and rubella, with a slight modification in the defined exposure periods of the diseases, and were approved by the CSTE in 2006. prep. by Dr. Fekadu .T 8/27/2021 261

Congenital Rubella Syndrome- The following case definition for congenital rubella syndrome (CRS) was approved by the CSTE in June 1999. Clinical case definition - An illness, usually manifesting in infancy, resulting from rubella infection in utero and characterized by signs or symptoms from the following categories: • Cataracts and congenital glaucoma, congenital heart disease (most commonly patent ductus arteriosus or peripheral pulmonary artery stenosis ), hearing impairment, pigmentary retinopathy • Purpura , hepatosplenomegaly , jaundice, microcephaly , developmental delay, meningoencephalitis , radiolucent bone disease Clinical description Presence of any defect(s) or laboratory data consistent with congenital rubella infection. Infants with CRS usually present with more than one sign or symptom consistent with congenital rubella infection. However, infants may present with a single defect. Hearing impairment is the most common single defect. Laboratory criteria for diagnosis • Isolation of rubella virus, or • Positive serologic test for rubella immunoglobulin M ( IgM ) antibody, or • Infant rubella antibody level that persists at a higher level and for a longer period than expected from passive transfer of maternal antibody (i.e., rubella titer that does not drop at the expected rate of a twofold dilution per month) • Detection of rubella-virus specific nucleic acid by polymerase chain reaction. prep. by Dr. Fekadu .T 8/27/2021 262

WHO Clinical Staging of HIV/AIDS for Infants and Children with Established HIV Infection   Clinical Stage 1 Asymptomatic Persistent generalized lymphadenopathy Clinical Stage 2 Unexplained persistent hepatosplenomegaly Papular pruritic eruptions ,Extensive wart virus infection ,Extensive molluscum contagiosum ,Recurrent oral ulcerations Unexplained persistent parotid enlargement Lineal gingival erythema Herpes zoster Recurrent or chronic upper respiratory tract infections (otitis media, otorrhoea , sinusitis, tonsillitis ) Fungal nail infections prep. by Dr. Fekadu .T 8/27/2021 263

Clinical Stage 3 Moderate unexplained malnutrition not adequately responding to standard therapy Unexplained persistent diarrhoea (14 days or more ) Unexplained persistent fever (above 37.5 intermittent or constant, for longer than one month) Persistent oral candida (outside first 6-8 weeks of life) Oral hairy leukoplakia Acute necrotizing ulcerative gingivitis/periodontitis Lymph node TB/ Pulmonary tuberculosis Severe recurrent presumed bacterial pneumonia Symptomatic lymphoid interstitial pneumonitis Chronic HIV-associated lung disease including bronchiectasis Unexplained anaemia (<8g/dl ), neutropenia (<500/mm3) or chronic thrombocytopenia(< 50,000/ mm3 ) prep. by Dr. Fekadu .T 8/27/2021 264

Clinical Stage 4 Pneumocystis pneumonia Recurrent severe presumed bacterial infections (e.g. empyema , pyomyositis , bone or joint infection, meningitis, but excluding pneumonia) Chronic herpes simplex infection; ( orolabial or cutaneous of more than one month's duration or visceral at anysite ) Extrapulmonary tuberculosis Kaposi sarcoma Oesophageal candidiasis (or candida of trachea, bronchi or lungs) Central nervous system toxoplasmosis (outside the neonatal period) HIV encephalopathy Cytomegalovirus (CMV) infection; retinitis or CMV infection affecting another organ, with onset at age over 1 month Extrapulmonary cryptococcosis including meningitis Disseminated endemic mycosis ( extrapulmonary histoplasmosis , coccidiomycosis , penicilliosis ) Chronic Cryptosporidiosis Chronic Isosporiasis Disseminated non- tuberculous mycobacteria infection Acquired HIV-associated rectal fistula Cerebral or B cell non-Hodgkin lymphoma Progressive multifocal leukoencephalopathy HIV-associated cardiomyopathy or HIV-associated nephropathy prep. by Dr. Fekadu .T 8/27/2021 265

18-Which of the following is the most common etiologic agent of urinary tract infection in children? A. Escherichia-Coli B. Streptococcus Species C. Staphylococcus aureus D. Klebsiella prep. by Dr. Fekadu .T 8/27/2021 266

19-Which of the following are the most common causes of acute otitis media in children? A. Strep. Pneumonia and H-influenza B. H-influenza and staph. aureus C. Strep. Pneumonia and staph.aureus D. H-influenza and pseudomonas argunosa prep. by Dr. Fekadu .T 8/27/2021 267

20-A 6 yr old boy is dx with acute rheumatic fever with the following findings ; migratory polyarthritis , fever, elevatead ESR, and evidence of preeceding streptococcal infection .which of the ff is the appropriate Secondary prophylaxis regarding the management of of the above pt? A-Amoxicillin for 7 day B- Aspirin C-benzathine penicillin G, 1.2 million U IM D- prednisolone prep. by Dr. Fekadu .T 8/27/2021 268

21-one of the following is not diagnostic investigation of HIV for 8 month infant A-DNA PCR B-RNA PCR C-Antibody test D-All are diagnostic prep. by Dr. Fekadu .T 8/27/2021 269

ARF Prevention Primary prophylaxis : Appropriate antibiotic therapy before the 9th day of symptoms of acute GAS pharyngitis is highly effective in preventing 1st attacks of ARF About 30% of patients with acute rheumatic fever do not recall a preceding episode of pharyngitis . Secondary prophylaxis: continuous antibiotic prophylaxis, which should begin as soon as the diagnosis of acute rheumatic fever has been made and immediately after a full course of antibiotic therapy has been completed. Benzanthine pencillin G 600,000IU IM for < 27kg 1.2million IU for >27kg Cotrimoxazole or Erythromycine / azithromax for pencillin allergy prep. by Dr. Fekadu .T 8/27/2021 270

prep. by Dr. Fekadu .T 8/27/2021 271

prep. by Dr. Fekadu .T 8/27/2021 272

prep. by Dr. Fekadu .T 8/27/2021 273

DNA PCR at 6 weeks or at earliest opportunity after age 6 weeks Start Cotrimoxazole prophylaxis ARV prophylaxis as per the national *PMTCT guideline   Infant or child remains WELL Continue follow- up; Continue Cotrimoxazole Rapid test at ≥12 months of age or at least 6 weeks after complete cessation of breastfeeding   Initiate ART If infant or child gets SICK Repeat DNA PCR Continue Cotrimoxazole   NEGATIVE Continue follow up per national guideline; continue Cotrimoxazole, ARV per national PMTCT guideline POSITIVE NEGATIVE HIV infection unlikely Look for other causes Rapid test at ≥12 months of age or >6 weeks after complete cessation of breastfeeding   POSITIVE   POSITIVE Repeat DNA PCR Initiat e ART NEGATIVE** NOT HIV-infected   Follow- up in routine child health service   HIV-infected Initiate ART , take DBS to repeat viral test Positive Negative Fo llow- up until HIV is excluded   *If the mother is on ART ensure infant ARV prophylaxis is only for 6 weeks **The child should not have been on breast feeding at least for 6 weeks before declaring HIV negative       HIV-exposed infant (Infant born to HIV-infected mother or HIV antibody positive infant <18 m onths of age) prep. by Dr. Fekadu .T 8/27/2021 274

22-A four year-old child was brought to a health institution with a complaint of cough for three weeks duration. On history, the child had fever and night sweats. Gene X -pert result suggested PTB positive. What is the other mandatory screening to be done for this child? (A) Assess for anemia (B) Assess for diarrhea (C) Assess for dehydration d-Assess for acute malnutrition prep. by Dr. Fekadu .T 8/27/2021 275

23-A 2yrs old 10 kg child is assessed as having some dehydration because he is irritable and has sunken eyes. How do you manage him? A-ORS 1000ml over 5hrs B-RL 750ml over 5hrs C-ORS 750ml over 4hrs D-RL 1000ml over 4hrs prep. by Dr. Fekadu .T 8/27/2021 276

24-A 6-month-old girl, comes in for her checkup with her mother and grandmother. A look in the girl’s mouth reveals no teeth. What is the usual age for the eruption of the primary teeth? (A) 2–4 months (B) 4–6 months (C) 6–8 months (D) 8–10 months (E) 10–12 months prep. by Dr. Fekadu .T 8/27/2021 277

25-The girl’s grandmother goes on to report that she is sure that her grandaughter will be left handed, just like her. Consistent use of a dominant hand is established at which of the following ages? (A) 12 months (B) 24 months (C) 4–6 years (D) 7 years (E) 9 years prep. by Dr. Fekadu .T 8/27/2021 278

26-A mother brings her 6-month-old son in for a checkup. She is concerned that when she is playing with him and puts a toy behind her back, he does not try to find it. Object permanence, the understanding that objects continue to exist even when not seen, is a major milestone that occurs closest to which of the following ages? (A) 9–12 months (B) 15 months (C) 18 months (D) 24 months (E) 30 months prep. by Dr. Fekadu .T 8/27/2021 279

27-A 4 -year-old child presents to your office with fever, and increased work of breathing manifested by tachypnea and retractions. Achest x-ray confirms lobar pneumonia. What is the most likely etiology of pneumonia in this child? (A) Mycoplasma pneumoniae (B) Streptococcus pyogenes (C) Chlamydophila pneumoniae (D) Streptococcus pneumoniae (E) Staphylococcus epidermidis prep. by Dr. Fekadu .T 8/27/2021 280

28-A 15-year-old female comes to your office with 4-day history of fever and cough. Chest x-ray findings include bilateral patchy infiltrates. Oxygen saturation is normal. What is the most likely etiology of her pneumonia? (A) Streptococcus pneumoniae (B) Staphylococcus aureus (C) Mycoplasma pneumoniae (D) Chlamydophila psittaci (E) Pneumocystis jiroveci prep. by Dr. Fekadu .T 8/27/2021 281

29-During the delivery room resuscitation of a vigorous term newborn, which of the following should be performed first? Verify the airway is clear, dry, and stimulate the infant. (B) The heart rate should be auscultated . (C) Breath sounds should be auscultated . (D) The mouth and trachea should be suctioned. (E) Assess color and administer oxygen if necessary. prep. by Dr. Fekadu .T 8/27/2021 282

Resuscitation of the newborn is a systematic procedure with a logical progression of events, which best ensures a successful outcome. The infant should be placed on a warmer, left uncovered, and positioned in the “sniffing” position. If no meconium is present, secretions should be removed from the mouth and then nose with a bulb syringe in order to clear the airway. Once the airway is clear, drying the infant thoroughly will provide stimulation for the infant and will prevent heat loss. Removal of the wet linen is an important step to prevent further heat loss. Assessment of heart rate, respirations, and color should not be completed until these important first steps have been carried out. Suction of the mouthand trachea may be indicated as a first step if meconium is present and the baby is not vigorous prep. by Dr. Fekadu .T 8/27/2021 283

30-A 7-day-old infant develops white, cheesy patches on the tongue and buccal mucosa with mild inflammation of the mucosa. Which of the following organisms is most likely the cause of these oral lesions? (A) Candida albicans (B) Listeria monocytogenes (C) Escherichia coli (D) group A streptococcus (E) group B streptococcu prep. by Dr. Fekadu .T 8/27/2021 284

The findings described are typical of thrush (oral infection with C albicans ), which is common in young infants. Although E coli , L monocytogenes , and group B streptococcus all are important pathogens in the neonatal period, they typically are not associated with pharyngeal infection or oral exudate . Group A streptococcus is a common cause of exudative tonsillitis in the older child but is an extremely rare pathogen in the newborn infant likely due to maternally acquired antibody. Additionally, the exudate noted with group A streptococcal infection would be in the area of the tonsils rather than on the buccal mucosa. prep. by Dr. Fekadu .T 8/27/2021 285

31-A 1700-g infant was asphyxiated at birth, after a successful resuscitation, the infant had numerous apneic episodes. On the third day of life, the infant began to vomit. Abdominal distention and bloody stools were noted. Which of the following is the most likely diagnosis? (A) congenital aganglionic megacolon (B) intussusception (C) necrotizing enterocolitis (D) Shigella enteritis (E) volvulus prep. by Dr. Fekadu .T 8/27/2021 286

Necrotizing enterocolitis (NEC) occurs frequently among low birth weight infants who have had repeated episodes of hypoxia or poor perfusion. The usual signs of NEC are abdominal distention, bloody stools, vomiting, hypothermia, and lethargy. Intussusception is rare in the neonatal period. Volvulus and congenital aganglionic megacolon ( Hirschsprung disease) are unrelated to low birth weight or hypoxia and usually are associated with failure to pass stool. Shigella infection is very uncommon in the nursery prep. by Dr. Fekadu .T 8/27/2021 287

32-At what age would one expect children to achieve to build a three or /4-cube tower and can point to her nose. A-6 weeks B. 6 months C. 18 months. D. 10 months E. 12 months prep. by Dr. Fekadu .T 8/27/2021 288

Developmental milestones NEONATE = Prone- turns head from side to side; head sags on ventral suspension, Supinegenerally flexed and a little stiff (fencing posture), Pulled to sit- Marked head lag, fixate face on light in line of vision, becomes alert in response to voice, Visual preference for human face - 1 MONTH= raises head slightly when prone, alerts to sound, regards face, moves extremities equally - 2-3 MONTHS= smiles, holds head up, coos, reaches for familiar objects, recognizes parent - 4-5 MONTHS= rolls front to back & back to front, sits well when propped, laughs, orients to voice, enjoys looking around, grasps rattle, bears some weight in legs prep. by Dr. Fekadu .T 8/27/2021 289

6 MONTHS= sits unsupported, transfers objects from hand to hand Uses whole hand as palmar grasp, feeds self-crackers, forms monosyllabic vowel sound- e. g. goo, dah , prefers mother and recognizes that someone is a stranger - 8-9 MONTHS= crawls, pulls to stand, pincer grasp, feeds self with bottle, sits without support, explores env’t , forms repetitive consonant sounds (“mama,” “dada”), Understands “no”, waves bye-bye - 12 MONTHS= Walks with one hand held, rises independently, takes several steps, says a few words besides “mama,” “dada, comes when called Cooperates with dressing, drinks from a cup, throws objects, imitates action - 15 MONTHS= Walks alone with uneven steps, crawls upstairs, follows simple commands, may name a familiar object (e.g. ball), Indicates some desires or needs by pointing, hugs parents - 18 MONTHS= runs stiffly, sits on small chair, walks upstairs with 1 hand held, speak 10 words (average), names pictures, identifies 1 or more parts of body, feeds self (uses cup and spoon), may complain when wet or soiled By 18 months can usually build a small tower of bricks and can point to several parts of the bod prep. by Dr. Fekadu .T 8/27/2021 290

33-Helen has a friend at nursery, and they enjoy playing with toy cars together. The approximate age at this point ? A- 3 months B. 6 Years C. 8 months D. 10 months E. 3-years. prep. by Dr. Fekadu .T 8/27/2021 291

36  MOnths Motor:Rides tricycle; stands momentarily on one foot Adaptive: Makes tower of 10 cubes; imitates construction of “bridge” of 3 cubes; copies circle; imitates cross Language: Knows age and sex; counts 3 objects correctly; repeats 3 numbers or a sentence of 6 syllables; most of speech intelligible to strangers Social: Plays simple games (in “parallel” with other children); helps in dressing (unbuttons clothing and puts on shoes); washes hands prep. by Dr. Fekadu .T 8/27/2021 292

N.B:At 3 years, children develop interactive play and turn-taking. It may emerge slightly sooner in those who attend nursery or in those with siblings. prep. by Dr. Fekadu .T 8/27/2021 293

34-At what age can an infant transfer objects from one hand to the other whilst sitting without support and with a straight back ? A-6 years B. 6 months C. 8 months. D. 10 months E. 12 months prep. by Dr. Fekadu .T 8/27/2021 294

Key:At 8 months of age Children transfer objects around 7 months of age. A rounded back requires control of T1 to T12, but a straight back requires T12 and L1 so develops later. A child will often ‘sit’ at 6 months but sit and be stable and with a straight back at 8 months prep. by Dr. Fekadu .T 8/27/2021 295

35-Lidy enjoys drawing. She has just learnt to copy drawing a square, and can build steps using blocks after being shown ? A-6 years B. 4 Years. C. 18 months D. 1 years E. 24 months prep. by Dr. Fekadu .T 8/27/2021 296

Asking a child to draw (copy) shapes is a good way of ‘estimating’ their age. They scribble first (age 2 years), then can copy a circle (by 3 years), and a square (by 4 years). A triangle (by 5 years) follows. prep. by Dr. Fekadu .T 8/27/2021 297

24-30 MONTHS= removes shoes, follows 2 step command, jumps with both feet, holds pencil knows first and last name, knows pronouns, points to body parts, runs, walk upstairs alternating feet 3 YEARS= dresses & undresses, walks up and down steps, draws a circle, makes full sentences, takes turns, knows age and sex, count 3 objects, wash hands & put shoe 4 YEARS= hops, skips, catches ball, memorizes songs, plays cooperatively, tells a story, knows colors, un buttons, draws cross & Square, throw ball over head 5 YEARS= jumps over objects, knows address & mother’s name, name 4 colors, count 10 objects, follows game rules, draws three-part man & triangle, hops on one food prep. by Dr. Fekadu .T 8/27/2021 298

Key :Asking a child to draw (copy) shapes is a good way of ‘estimating’ their age. They scribble first (age 2 years), then can copy a circle (by 3 years), and a square (by 4 years). A triangle (by 5 years) follows. prep. by Dr. Fekadu .T 8/27/2021 299

36-Mohammed, aged 8 months, has been vomiting and off his feeds for 2 days. Initially, he had episodes of crying uncontrollably, drawing his legs up into his abdomen as if in pain, and appeared fractious. His mother gave him some oral rehydration solution, but his vomiting continued and he has become lethargic. On admission to hospital he is in shock. What is the most likely diagnosis? Select one answer only. A. Gastroenteritis B. Intussusception . C. Malrotation and volvulus D. Meckel diverticulum E. Strangulated hernia prep. by Dr. Fekadu .T 8/27/2021 300

Key:A -Gastroenteritis Less likely, because episodes of crying uncontrollably with drawing of legs up into his B. Intussusception Correct. Intussusception is the most likely cause of the pain and shock. Although this could be a strangulated hernia, this should be evident on clinical examination. Follow an Airway, Breathing, Circulation approach, get senior help and speak to the radiologist. The diagnosis might be obvious on ultrasound. C. Malrotation and volvulus Less likely, because episodes of crying uncontrollably with drawing of legs up into his abdomen as if in pain are characteristic of intussusception . Bile-stained vomiting is often present in malrotation . However, the diagnosis must be considered. D. Meckel diverticulum Meckel diverticulum tends to present with bleeding per rectum as well as abdominal pain. Blood loss is rarely so severe to result in shock. E. Strangulated hernia Although this could be a strangulated hernia, this should be evident on clinical examination prep. by Dr. Fekadu .T 8/27/2021 301

37-Mohammed, aged 8 months, presented with the clinical scenario described in ques. He weighs 8 kg. He needs a bolus of normal saline 0.9% to treat his shock. What volume of fluid would you give initially? Select one answer only. A. 40 ml B. 160 ml. C. 320 ml D. 680 ml E. 800 ml prep. by Dr. Fekadu .T 8/27/2021 302

Key:160 ml Correct. This is 20 ml/kg initially, repeated as necessary. N.B:read initial fluid resuscitation in shock. prep. by Dr. Fekadu .T 8/27/2021 303

38-You are called to see a 3-year-old boy with a high fever. The nurse is worried that he is very sleepy. As you walk into the resuscitation room he makes no spontaneous response. You try calling his name but he makes no response. On stimulation, his eyes open, he cries and he raises his hand and pushes your hand away. What is this child’s Glasgow Coma Score (GCS)? Select one answer only. A. 8 B. 9. C. 10 D. 11 E. 12 prep. by Dr. Fekadu .T 8/27/2021 304

prep. by Dr. Fekadu .T 8/27/2021 305

KEY: B. 9 Correct. The Glasgow Coma Scale is made up of three parts. Best motor response (a score of 1–6 is possible); Best verbal response (a score of 1–5 is possible) and Best eye opening (1–4 is possible). Here the child scores 5 for best motor response (localizes pain) and \he raises his hand and pushes and 2 each for best verbal and best eye response prep. by Dr. Fekadu .T 8/27/2021 306

39-Markan , , aged 8 months, has been vomiting and off his feeds for 2 days. Initially, he had episodes of crying uncontrollably, drawing his legs up into his abdomen as if in pain, and appeared fractious. His mother gave him some oral rehydration solution, but his vomiting continued and he has become lethargic. On admission to hospital he is in shock ,presented with the clinical scenario . He weighs 8 kg. He needs a bolus of normal saline 0.9% to treat his shock. What volume of fluid would you give initially? Select one answer only. A. 40 ml B. 160 ml C. 320 ml D. 680 ml E. 800 m prep. by Dr. Fekadu .T 8/27/2021 307

KEY:1600 ml Correct. fluid requirement is calculated by adding • Deficit: 10% of 8 kg = 800 ml • Maintenance: 100 ml/kg per 24 hours = 800 ml • Continuing losses: 0 ml Total = 1600 ml Paediatrics for maintenance fluid requirements at different weights. prep. by Dr. Fekadu .T 8/27/2021 308

40-Roble, aged 10 months, is rushed to the children’s emergency department after being found submerged in the bath. His mother runs screaming into the department saying ‘Help my baby, please’. Which is the next most appropriate step? Select one answer only. A. Commence chest compressions in a ratio of 15 : 2 B. If the child is not breathing, commence bag and mask ventilation C. Place the child onto the examination couch and put his head into the neutral position D. Remove wet clothing/towels and dry the baby vigorously E. Stimulate the baby and shout for help. prep. by Dr. Fekadu .T 8/27/2021 309

KEY:Stimulate the baby and shout for help Correct. All resuscitation algorithms ensure that the patient is assessed in a sequential manner, adopting an Airway, Breathing and Circulation approach. The paediatric life support algorithm states that you: Check safety (not strictly necessary within hospital), stimulate and shout for help. Calling for help early on in these situations is paramount as you need many people for resuscitation prep. by Dr. Fekadu .T 8/27/2021 310

41-You are called to the resuscitation room where there is a 6-year-old child who has arrived by ambulance. The child has been having a generalized seizure for 15 min. The ambulance crew gave a dose of buccal midazolam 5 min ago.The emergency doctor has maintained the airway and has applied oxygen. His capillary refill time is less than 2 seconds and his heart rate 120 beats/min. What is the next most appropriate management step? Select one answer only. A. Administer further anticonvulsant B. Check blood glucose level. C. Gain intravenous access D. Request senior review E. Start bag and valve mask ventilation prep. by Dr. Fekadu .T 8/27/2021 311

KEY:Check blood glucose level -Correct. This is the most appropriate next step as, if the patient is hypoglycaemic , the only treatment to stop the seizure would be to administer glucose. Ideally, intravenous glucose will be given but if access is not achieved, then glucose gel buccally Administer further anticonvulsant After 10 minutes it is recommended to give a further dose of an anticonvulsant if still having a seizure C- Gain intravenous access It is difficult to do this whilst the child is having a seizure. It can be very helpful but there are more important treatment steps. This is often done at the same time by other team members. D. Request senior review It is important to make an ABC plus d on’t e ver f orget g lucose (ABCDEFG) assessment yourself before seeking senior review. E. Start bag and valve mask ventilation Although it can be difficult to assess breathing in a child with seizures, it is usually sufficient to administer oxygen unless breathing stops completely. This can occur after benzodiazepine therapy but is uncommon after a single dose prep. by Dr. Fekadu .T 8/27/2021 312

42-Biruk , a 3-year-old girl, was at a village fete. She suddenly developed swollen cheeks and lips and a widespread urticarial rash. She is rushed to the nearby general practice surgery, where it is noted that her breathing is very noisy. She is distressed and frightened. On auscultation she has widespread wheeze. Which medication would you give first? A. Intramuscular adrenaline. B. Intramuscular antihistamine C. Intravenous hydrocortisone D. Oral antihistamine E. Oral corticosteroid prep. by Dr. Fekadu .T 8/27/2021 313

KEY:A. Intramuscular adrenaline Correct. - This child has anaphylaxis, which is life threatening as she has both upper airway obstruction (noisy breathing) and bronchoconstriction (wheeze). -Either of these on their own or any signs of shock would be enough to constitute a diagnosis of anaphylaxis. -Priority is to manage the airway and give oxygen via a non- rebreathe mask. -The first medication to give would be intramuscular adrenaline. Intramuscular antihistamine This would make the situation worse, as it is painful and will not directly treat the upper airway obstruction. prep. by Dr. Fekadu .T 8/27/2021 314

43-Daniel, age 10 years, has diabetes mellitus and has been playing football at his friend’s house. He has been brought to the emergency department as he has become confused and is sweaty. He walks into the department . T he clinical features of Daniel are suggestive of ? A- Hypoglycaemia . B-Seizure Disorder C-Trauma D-Infection prep. by Dr. Fekadu .T 8/27/2021 315

44-Alem is a 2-month-old baby who is seen in the paediatric outpatient department. She was born at term, weighing 3.5 kg and is breastfed. Her mother is concerned as she has vomited some of the milk after most feeds since birth. She cries when she vomits. She is continuing to grow along the 50th centile.What is the most likely diagnosis? Select one answer only. A. Gastro- oesophageal reflux. B. Helicobacter pylori infecti C- Infant colic D. Overfeeding E. Pyloric stenosis prep. by Dr. Fekadu .T 8/27/2021 316

KEY : A-Gastro- oesophageal reflux Correct. Gastro- oesophageal reflux is caused by the involuntary passage of gastric contents into the lower oesophagus . These infants can vomit several times per day but still continue to gain weight appropriately. If complications are present, it is called gastro oesophageal reflux disease and needs treatment. B. Helicobacter pylori infection : presents with abdominal pain, typically in an older child. C. Infant colic Whilst this is common, it is typified by pain (and crying) in the early evening lasting for a few hours. Only a minority of cases are thought to be due to gastro- oesophageal reflux. D. Overfeeding Whilst this is a common cause of vomiting, it is uncommon in breastfed babies. The clue in an examination will be that the feed volumes are significantly higher than you would expect. For a bottle fed infant, the feed volume will often be in excess of 200 ml/kg per day. prep. by Dr. Fekadu .T 8/27/2021 317

45-Abel is 5 years old and has had an intermittent fever for 4 weeks. He presents to the Emergency Department with a headache and neck stiffness. A CT scan is performed, which is normal. A lumbar puncture is performed. You receive the following result from the laboratory: • cerebrospinal fluid (CSF) microscopy: 95 lymphocytes, 10 neutrophils and 0 red blood cells/mm3 • CSF protein: 2.2 g/L------=== • CSF glucose: 1.3 mmol /L=23mg/dl • blood glucose: 6.3 mmol /L =115mg/dl What is the most likely diagnosis? Select one answer only. A-Ascending polyneuritis ( Guillain–Barrésyndrome ) B- Bacterial meningitis C. Blood-stained tap D. Tuberculosis meningitis E. Viral meningitis prep. by Dr. Fekadu .T 8/27/2021 318

KEY :A. Ascending polyneuritis ( Guillain–Barrésyndrome ) In ascending polyneuritis ( Guillain–Barré syndrome), the protein level would also be high but the white cells would not be raised and the glucose would not be so low. It is important to ensure that a paired blood sample is taken for serum glucose. B. Bacterial meningitis The white cell count shows predominantly Lymphocytes, and the neutrophil count is only marginally raised. C. Blood-stained tap No red blood cells are reported in this sample. D. Tuberculosis meningiti Correct. The white cell count shows predominantly lymphocytes. This could also be seen in viral meningitis, but the very markedly raised protein and very low glucose in the CSF together with the clinical history are suggestive of tuberculosis. E. Viral meningitis The white cell count shows predominantly lymphocytes. This could also be seen in viral meningitis, but the very markedly raised protein and very low glucose in the CSF together with the clinical history are suggestive of tuberculosis prep. by Dr. Fekadu .T 8/27/2021 319

prep. by Dr. Fekadu .T 8/27/2021 320

45-Regarding the HDN ( hemorragic diseases of the new born ) all of the ff are true, except? A.Age of onset was usually in2_7 days old newborn B.more frequent in breast-fed than in formula-fed infants C.bleeding time is normal D .oral vitamin K is as effective as intramuscular route in the preventiOn and Treatment of the disease . E.prothrombin time (PT ), and partial thromboplastin time (PTT ) are prolong prep. by Dr. Fekadu .T 8/27/2021 321

KEY . D_Although oral vitamin K ( birth, discharge, 3-4 wk: 12 mg has been suggested as an alternative, oral vitamin K is less effective in preventing the late onset of Bleeding due to vitamin K deficiency and thus cannot be recommended for routine therapy . The intramuscular route remains the method of choice . prep. by Dr. Fekadu .T 8/27/2021 322

46-A 12 year-old brought in to the hospital with a clinical sign and symptoms of polydypsia , polyphagia , and polyuria and weight loss. After detailed diagnosis, the physician recommends lifelong insulin therapy. Which type of diabetes mellitus is most likely diagnosed? A-Gestational Diabetes Mellitus B-Secondary Diabetes Mellitus C-Type I Diabetes Mellitus. D-Type II Diabetes Mellitus prep. by Dr. Fekadu .T 8/27/2021 323

47.A Physician ordered 1000ml of normal saline to run over 6 hours. What is the drop rate per minute to administer the fluid? A-41 drop/min B-42 drop/min C-52 drop/min D-56 drop/min prep. by Dr. Fekadu .T 8/27/2021 324

Drop rate can be calcul Total volume x20/time period in hrx60 prep. by Dr. Fekadu .T 8/27/2021 325

47-Which of the following are the leading causes of neonatal mortality in Ethiopia? A- Pretem , jaundice and malnutrition B-Diarrhea, malnutrition and asphyxia C-Preterm, infections and asphyxia. D-Infections, sepsis and diarrhea prep. by Dr. Fekadu .T 8/27/2021 326

49-Which of the following is true about pathological jaundice? A-Appear after 24 hours of life B-Appear within 24 hours of life. C-Serum of bilirubin <12 mg / dl D-Serum bilirubin <5 mg / dl/day prep. by Dr. Fekadu .T 8/27/2021 327

prep. by Dr. Fekadu .T 8/27/2021 328

50-What is the recommended time to begin complementary feeding to infants? A. At 4 to 5 months of age B. At 6 months of age. C. When the birth weight has tripled D. When tooth eruption has started prep. by Dr. Fekadu .T 8/27/2021 329

51-Which of the following is a sign of good attachment? A. Mouth wide open . B. Baby facing towards the mother C. Whole body supported D. Baby close to the mother prep. by Dr. Fekadu .T 8/27/2021 330

52-Which of the following is not true about acute rheumatic fever in children? A.It is common cause of acquired heart disease in children. B. Acute rheumatic fever is preventable C. Proper treatment of streptococcal pharyngitis can eliminate its risk D. Peak age for acute rheumatic fever is up to 3 years. prep. by Dr. Fekadu .T 8/27/2021 331

53-A female infant, is delivered by caesarean section at 32 weeks’ gestation because of maternal pre- eclampsia . Her birth weight is 1.9 kg. No resuscitation is required. At 2 hours of age she develops respiratory distress, with a respiratory rate of 70 breaths/min, grunting respirations, and indrawing of her rib cage. Respiratory support with CPAP (continuous positive airway pressure) and 45% oxygen is required. A chest X-ray is taken at 4 hours of age, and is show that a diffuse granular or ‘ground glass’ appearance .the most likely reason that this baby needs oxygen therapy and respiratory support? Select one answer only. A. Aspiration of meconium has resulted in lung collapse B. Blood is still flowing from the pulmonary artery to the aorta as in the fetal circulation C. The alveoli still contain fluid D. The chest wall and ribs are too compliant E. There is ventilation-perfusion mismatch from surfactant deficiency. prep. by Dr. Fekadu .T 8/27/2021 332

Aspiration of meconium has resulted in lung collapse Meconium aspiration is associated with term or post-term infants. There are patchy changes on chest X-ray (CXR). B. Blood is still flowing from the pulmonary artery to the aorta as in the fetal circulation Some blood may ‘shunt’ across a PDA (persistent ductus arteriosus ) but this would usually flow from the high pressure aorta into the pulmonary artery as pulmonary vascular resistance should drop after birth and inflation of the lungs. prep. by Dr. Fekadu .T 8/27/2021 333

C. The alveoli still contain fluid Retention of a small amount of fluid is common and results in transient tachypnoea of the newborn. D. The chest wall and ribs are too compliant The ribs are compliant but this is not the main reason for respiratory distress. E. There is ventilation-perfusion mismatch from surfactant deficiency Correct. Surfactant deficiency is common in preterm infants. This results in alveolar collapse which in turn results in areas of lung being perfused but not ventilated. The CXR appearance here is typical with a diffuse granular or ‘ground glass’ appearance prep. by Dr. Fekadu .T 8/27/2021 334

54-Robel is a full term male infant, born 10 hours ago. His mother is blood group O rhesus positive and her membranes ruptured 2 days before delivery. He is breastfeeding well but the midwife noticed he looks jaundiced. On examination the baby is clinically well but markedly jaundiced. What investigation should be performed first? Select one answer only. A. Bilirubin level. B. Blood culture C. Blood group D. Congenital infection screen E. Direct antibody test prep. by Dr. Fekadu .T 8/27/2021 335

Bilirubin level Correct. Jaundice starting at less than 24 hours of age is most likely due to haemolysis and may rapidly rise to dangerously high levels. It needs urgent assessment and close monitoring. The most urgent investigation is to measure the bilirubin level, as this will determine the management required. B. Blood culture Blood cultures are taken to exclude infection but do not influence immediate management. C. Blood group Blood group will inform if ABO or Rhesus incompatibility is a likely cause but will not influence immediate management. D. Congenital infection screen Congenital infection can cause early jaundice but there are usually clinical features and the jaundice is mild. E. Direct antibody test Direct antibody test is positive with rhesus disease and ABO incompatibility. prep. by Dr. Fekadu .T 8/27/2021 336

55-You are asked to review a newborn baby who is only 24 hours old and has developed very swollen eyelids with a purulent discharge. What management is required? Select one answer only. A. Clean with cool boiled water B. Intravenous antibiotics. C. Oral antibiotics D. Reassure that it will resolve spontaneously E. Topical antibiotic therapy prep. by Dr. Fekadu .T 8/27/2021 337

Clean with cool boiled water The eyes should be cleaned and carefully examined but also treated pending the results of any swabs. B. Intravenous antibiotics Correct. Purulent discharge and eyelid swelling in the first 48 hours of life must be taken seriously. This should be treated promptly, e.g. with a third-generation cephalosporin intravenously, as permanent loss of vision can occur. The most common cause of severe neonatal purulent conjunctivitis is Chlamydia trachomatis , but gonococcus may also be the cause. prep. by Dr. Fekadu .T 8/27/2021 338

C. Oral antibiotics Oral antibiotic absorption in the newborn is highly variable and therefore cannot be relied upon to treat this (or other) infections. D. Reassure that it will resolve spontaneously Whilst incomplete canalization of the nasolacrimal duct commonly leads to sticky eyes, the discharge is rarely purulent and the eyes are not swollen. E. Topical antibiotic therapy Topical antibiotic therapy is often prescribed for a sticky eye in older infants and children but is insufficient in this situation prep. by Dr. Fekadu .T 8/27/2021 339

56- Heln was born at term weighing 4 kg. At 6 hours of age she was noted to be breathing fast and have a low temperature. She was born by normal vaginal delivery and the membranes had ruptured 24 hours previously. has not breastfed since birth and has had one vomit. On P/exam, she is lethargic and her core temperature is 35.5° C. She has a respiratory rate of 90 breaths/min, her central capillary refill time is 4 seconds, pulse 180/min and oxygen saturation is 89% in air. Her chest X-ray shows consolidation at the right base What is the most likely causative organism for her infection? A. Escherichia coli (E. coli) B. Group B streptococcus. C. Herpes simplex virus (HSV) infection D. Listeria monocytogenes E. Staphylococcus aureus prep. by Dr. Fekadu .T 8/27/2021 340

Neonatal sepsis Neonatal sepsis is defined as a clinical syndrome of bacteremia with systemic signs and symptoms of infection in the first 4 weeks of life. Neonatal infection is one of the major causes of mortality and morbidity. Bacterial sepsis and meningitis often are linked closely in neonates; meningitis is present with Early-onset sepsis (in 30% of cases), late onset sepsis (in 75% of cases) Classification Early-onset infections are acquired before or during delivery. Late-onset infections are acquired after delivery in the normal newborn nursery, neonatal intensive care unit (NICU), or the community. 1. Early Sepsis (Birth to 7 days, usually less than 72 hrs ) 2. Late Onset Sepsis (7 - 30 days) 8/27/2021 prep. by Dr. Fekadu .T 341

Etiology Early Onset Neonatal Sepsis (EONS): Group B Streptococcus (GBS) E. Coli Listeria monocytogenes Streptococcus species ie . Viridans Due to maternal (maternal flora) or perinatal factors

Etiology Late Onset Neonatal Sepsis (LONS): Staphylococcus aureus Coagulase -negative staphylococcus Gram negative bacilli ie . E.coli,Klebsiella Listeria monocytogenes Nosocomial or focal infection

Pathogenesis of EONS Vertical transmission of bacterial agents that infect the amniotic fluid and/or vaginal canal may occur in utero or, more commonly, during labor and/or delivery. Most of the time the fetus or neonate is not exposed to potentially pathogenic bacteria until the membranes rupture and the infant passes through the birth canal and/or enters the extrauterine environment. The human birth canal is colonized with aerobic and anaerobic organisms that may result in ascending amniotic infection and/or colonization of the neonate at birth.

Chorioamnionitis Clinical syndrome of intrauterine infection, ( maternal fever, with /without local or systemic signs of chorioamnionitis (uterine tenderness, foul-smelling vaginal discharge/amniotic fluid, maternal leukocytosis , maternal and/or fetal tachycardia )). Results from microbial invasion of amniotic fluid, often due to prolonged rupture of the chorioamniotic membrane. >or =18 hr. May also be asymptomatic, dxed only by amniotic fluid analysis or pathologic examination of the placenta. EOS increases from 1% - 4%

Pathogenesis of Late-Onset Postnatal Infections  Late-onset infections can be acquired by the following two mechanisms: Maternal vertical transmission: resulting in initial neonatal colonization that evolves into later infection. Horizontal transmission : direct contact with care providers or environmental sources. Disruption of the intact skin or mucosa, which can be due to invasive procedures ( eg , intravascular catheter), increases the risk of late-onset infection. Most cases of meningitis result from hematogenous dissemination.

Risk factors for Neonatal Sepsis prematurity, Male sex underlying illness Five minute Apgar score ≤6 invasive procedures, inoculum size, Multiple gestation virulence of the infecting organism ROM>or = 18 hours risk of proven sepsis 10-x to 1 % genetic predisposition, the innate immune system, host response, and transplacental maternal antibodies . Resuscitation at birth, * Bacterial colonization does not always result in disease.

Clinical Manifestations Aspiration or ingestion of bacteria in amniotic fluid congenital pneumonia or systemic infection manifestations become apparent before delivery (fetal distress, tachycardia), at delivery (failure to breathe, respiratory distress, shock), or after a latent period of a few hours (respiratory distress, shock). Aspiration or ingestion of bacteria during the birth process infection after an interval of 1-2 days.

Clinical Manifestations Non specific GENERAL : Fever, temperature instability, “Not doing well”, Poor feeding GASTROINTESTINAL SYSTEM :Abdominal distention    Vomiting, Diarrhea, Hepatomegaly RESPIRATORY SYTEM : Apnea, dyspnea Tachypnea , retractions,   Flaring, grunting,   Cyanosis RENAL SYSTEM  :  Oliguria CARDIOVASCULAR SYSTEM  :  Pallor; mottling; cold clammyskin , Tachycardia,   Hypotension,   Bradycar-dia   

Investigations CBC with differential: should be obtained after 4 hrs of life. Blood cultures: positive within 36 hours,90% sensitive Urine cultures: LONS Lumbar puncture : useful in clinically ill newborns or those with positive blood cultures Tracheal aspirates

Ixs Platelet count :Late sign and very nonspecific Acute phase reactants CRP rises early, monitor serial values (greater than 1.0 mg/ dL is 90 percent sensitive) ESR rises late CXR: In infants with respiratory symptoms Difficult to distinguish GBS or Listeria pneumonia from uncomplicated RDS Renal ultrasound and/or VCUG : infants with accompanying UTI

Management Maternal GBS prophylaxis Supportive :ABCDs of life Empiric Antibiotic Therapy -Penicillins and Aminoglycosides

Treatment: General supportive measures, including respiratory and hemodynamic management, are combined with antibiotic treatment. For early onset (less than 7 days) Antibiotic – Ampicillin and Gentamycin Duration: If positive cultures – minimum 7 days For late onset (7-30 days) Antibiotic – Ampicillin and Gentamicin - In certain cases were patient is critically sick or staphylococcal infection is likely ( pustular skin rash, osteomyelitis …) start with triple antibiotics ( cloxacillin , ampicillin and gentamycin ) - If no improvement after 48 hours, or the infant’s condition worsens. Consider changing antibiotics to: Cloxacillin , ceftriaxone and gentamicin or vancomycine and gentamicin 8/27/2021 prep. by Dr. Fekadu .T 353

Treatment of neonatal sepsis with meningitis - Antibiotics the same as for sepsis but with higher dose and prolonged duration ( Gentamycin for two weeks the rest for three weeks) . 8/27/2021 prep. by Dr. Fekadu .T 354

8/27/2021 prep. by Dr. Fekadu .T 355

A. Escherichia coli (E. coli) An important cause of neonatal sepsis but not as common as Group B streptococcus. Antibiotic regimens to treat newborn infants must cover this organism though. B. Group B streptococcus Correct. The most likely to have early onset sepsis and this can be caused by infection in the chest, urine or cerebrospinal fluid (meningitis). The most common organism causing early-onset sepsis is Group B streptococcus . Listeria monocytogenes , E. coli, and S taphylococcus aureus may cause early-onset sepsis, but are less common causes. C. Herpes simplex virus (HSV) infection HSV infection in the neonate is rare but very serious when it occurs. It can present with localized lesions, encephalitis, or disseminated disease. Skin lesions may not be present. Treatment is with intravenous aciclovir . Primary maternal HSV infection may not be diagnosed at delivery. D. Listeria monocytogenes An important cause of neonatal sepsis and meningitis regimens to treat newborn infants must cover this organism though prep. by Dr. Fekadu .T 8/27/2021 356

57- was born at 39 weeks’ gestation by elective caesarean section because of pre- eclampsia . His birth weight was 3.7 kg. His mother breastfed him as soon as she had recovered from the general anaesthetic . He fed well but is vomiting after every feed. He is now 18 hours old and after the last 2 feeds he ‘vomited everything up’ and it was greenish in colour . On examination his temperature is 36.5° C and he is alert and hungry. His abdomen is not distended. He has not yet passed meconium . What is the most likely diagnosis? Select one answer only. A. Duodenal atresia . B. Hirschprung disease C. Meconium ileus D. Neonatal sepsis E. Pyloric stenosis prep. by Dr. Fekadu .T 8/27/2021 357

Duodenal atresia Correct. This is the most likely cause of persistent bilious vomiting on the 1st day of life. Malrotation and volvulus can also cause this presentation, but the vomit may contain blood and the abdomen may be tender. A plain abdominal X-ray would be helpful as it may show the classic ‘double bubble’. B. Hirschprung disease Affects the rectum and sometimes the colon, therefore causes distal bowel obstruction causing marked abdominal distension. C. Meconium ileus Affects the lower ileum, so there would be abdominal obstruction and therefore distension. D. Neonatal sepsis Neonatal sepsis can cause vomiting, which is sometimes slightly bile-stained, but there are no risk factors from labour and delivery, and no other features on clinical examination. E. Pyloric stenosis the vomit is not bile-stained as the obstruction is above the ampulla of Vater and it presents at about 6 weeks of age prep. by Dr. Fekadu .T 8/27/2021 358

8/27/2021 prep. by Dr. Fekadu .T 359 A plain abdominal X-ray would be helpful as it may show the classic ‘double bubble’.

58-a full-term male infant, with a birth weight of 3.7 kg, is born by elective caesarean section. His mother was well during pregnancy and had a normal blood glucose screen. becomes tachypnoeic with indrawing between his ribs at 2 hours of age. Examination is otherwise normal. A chest X-ray looks normal the most likely ddx ? A-Respiratory distress syndrome b- Tracheo - oesphageal fistula C-Transient tachypnoea of the newborn . D- Meconium aspiration prep. by Dr. Fekadu .T 8/27/2021 360

Transient tachypnoea of the newborn The most common cause of respiratory distress in a term infant. It is due to a delay in the reasorption of lung liquid – an increased risk following caesarean section, as the babies have not had to undergo the same physical and physiological stressors as those who pass through the birth canal(duration less than 24hr) Hyaline membrane disease (RDS type 1) Primarily, it is caused by immaturity of the lung (lack of adequate surfactant substance, which prevents collapse of alveoli at the end of expiration). prep. by Dr. Fekadu .T 8/27/2021 361

HMD /RDS Epidemiology Most cases of hyaline membrane disease occur in babies born before 37 weeks of gestation. Incidence is inversely related to gestational age and birth weight. It is uncommon in full term babies. The incidence based on gestational age is as follows:  Less than 28 weeks 60 – 80%,  32-36 weeks 15-35% in  >37 weeks 5%. Uncomplicated course characterized by peak severity at 1-3 days. Onset of recovery is at 72 hrs. Risk factors:  Low gestational age, low birth weight, Male predominance, maternal diabetes, perinatal asphyxia, elective caesarian section Clinical manifestations  Respiratory distress (Grunting, flaring, retraction, tachypnea )  Auscultatory findings – markedly decreased air entry bilaterally  Cyanosis Investigation  CBC, chest X-ray, if possible blood gas analysis, septic work up, oxygen saturation Prevention  Antenatal corticosteroids (at least 24-48 hrs before delivery) given to pregnant women < 34 weeks of gestational period 8/27/2021 prep. by Dr. Fekadu .T 362

Oesophageal atresia (EA) is a congenital abnormality of the oesophagus that is caused by incomplete embryonic compartmentalization of the foregut. EA commonly occurs with a tracheo-oesophageal fistula (TEF) A : Esophageal atresia with distal tracheoesophageal fistula.  B : Esophageal atresia alone.  C : Esophageal atresia with proximal tracheoesophageal fistula.  D : Esophageal atresia with double fistula.  E : Isolated tracheoesophageal fistula. EA, esophageal atresia ; TEF, tracheoesophageal fistula. (Source: 8/27/2021 prep. by Dr. Fekadu .T 363

8/27/2021 prep. by Dr. Fekadu .T 364

59-Bini is a 6-year-old boy who is seen in the paediatric emergency department. He has been vomiting and has had diarrhoea for 3 days. His stool is watery and foul smelling but has no blood in it. P/ Examination reveals mild dehydration but is otherwise normal. What is the most likely organism that has caused his symptoms? A. Campylobacter B. Escherichia coli C. Giardia lamblia D. Rotavirus. E. Shigella prep. by Dr. Fekadu .T 8/27/2021 365

A.Campylobacter The most common cause of bloody diarrhoea B. Escherichia coli / Enterohemoragic e-coli/EHEC/ This occurs in outbreaks. Verotoxin -producing strains can result in haemolytic uraemic syndrome. C. Giardia lamblia - is a parasitic infection and is usually acquired whilst travelling abroad, although there are rare case. Persistent diarrhoea would warrant testing for this organism. prep. by Dr. Fekadu .T 8/27/2021 366

D. Rotavirus Correct. Rotavirus is a common cause of foul-smelling, watery diarrhoea . Escherichia coli , Shigella , and Campylobacter also cause explosive watery diarrhoea but can be associated with blood in the stools and in the UK are much less common than rotavirus. Prevention -introduction of rotavirus vaccine into the standard childhood immunization schedule. E. Shigella Consider this if there is blood in the stool prep. by Dr. Fekadu .T 8/27/2021 367

60-Alem aged 9 months, has had a 3 day history of diarrhoea and vomiting. On examination he is found to be quiet but alert, is tachypnoeic , has a tachycardia but normal pulses, dry mouth, no mottling of the skin but reduced skin turgor and a sunken fontanelle . Capillary refill time is 2seconds. His blood pressure is normal for his age. He continues to vomit even with oral rehydration solution given via a nasogastric tube. Alem plasma sodium is found to be 156 mmol /L (normal range, 135–145 mmol /L). He needs fluid as he has clinical dehydration. How would this fluid best be replaced? Select one answer only. A. Immediate bolus of 20 ml/kg of 0.9% sodium chloride followed by reassessment and replacement of remaining deficit over 24 hours with 0.9% sodium chloride solution B. Rehydration over 6 hours followed by repeat urea and electrolyte measurement and maintenance fluid only for a further 18 hours C. Rehydration over 24 hours with 0.18% sodium chloride/5% glucose solution D. Rehydration over 24 hours with 0.9% sodium chloride/5% glucose solution E. Rehydration over 48 hours with 0.9% or 0.45% saline . prep. by Dr. Fekadu .T 8/27/2021 368

A. Immediate bolus of 20 ml/kg of 0.9% sodium chloride followed by reassessment and replacement of remaining deficit over 24 hours with 0.9% sodium chloride solution A bolus of intravenous fluid is not warranted as he is not in shock. Although the pulse rate is not provided, a normal capillary refill time and normal blood pressure confirm this. B. Rehydration over 6 hours followed by repeat urea and electrolyte measurement and maintenance fluid only for a further18 hours As he has hypernatraemic dehydration, rapid rehydration should not be given to avoid potential brain damage from cerebral oedema . This is therefore not a safe plan. prep. by Dr. Fekadu .T 8/27/2021 369

C. Rehydration over 24 hours with 0.18% sodium chloride/5% glucose solution Rapid rehydration should be avoided to avoid potential brain damage from cerebral oedema . In this case the use of hypotonic sodium chlorid solution is dangerous as a rapid fall in serum sodium increases this child’s risks. The use of 0.18% sodium chloride solutions is now restricted D. Rehydration over 24 hours with 0.9% sodium chloride/5% glucose solution In most circumstances rehydration over 24 hours is appropriate; however, in hypernatremic dehydration it is important to make the correction more slowly. E. Rehydration over 48 hours with 0.9% or 0.45% saline Correct. Ben has hypernatremic dehydration, so rehydration must be slow, i.e. over 48 hours to avoid cerebral oedema . In hypernatremic dehydration the brain cells are contracted as water has moved to the extracellular compartment, as this has a higher osmolality . Changes in extracellular osmolality need to be slow to avoid rapid expansion of cells in the brain. Hypernatremic dehydration is clinically more difficult to detect than other forms of dehydration on clinical grounds prep. by Dr. Fekadu .T 8/27/2021 370

61-Which of the following condition is the most common route of transmission of neonatal tetanus? A.Injection sites B.Umbilical stamp . C.Aseptic procedure D.Circumcision prep. by Dr. Fekadu .T 8/27/2021 371

62-Biruk is a 10-month-old boy from the UK who presents to the Emergency Department with a 2-day history of fever and runny nose. He has been otherwise well. During the night he gradually developed a barking cough in association with a loud noise on inspiration. On examination he has a temperature of 38° C and noisy inspiration accompanied by marked sternal recession. His capillary refill time is normal. Select the ONE most likely diagnosis from the list below. A. Acute epiglottitis B. Anaphylaxis C. Bronchiolitis D. Laryngeal foreign body E. Laryngotracheobronchitis (croup). prep. by Dr. Fekadu .T 8/27/2021 372

Acute epiglottitis Epiglottitis would present with a shorter history, and the child would be extremely unwell (toxic). It is also very rare in countries with Haemophilus influenzae type b immunization. B. Anaphylaxis Anaphylaxis may cause stridor but also causes other symptoms, often including an urticarial rash. It would not cause a fever. C. Bronchiolitis Bronchiolitis causes wheeze and crepitations rather than stridor as in this child. D. Laryngeal foreign body The child has a fever and a gradual onset of symptoms. E. Laryngotracheobronchitis (croup) Correct. Laryngotracheobronchitis (croup) is mucosal inflammation and increased secretions affecting the airway. Croup occurs from 6 months to 6 years of age. Fever suggests infection. Inspiratory noises suggest upper airway obstruction. prep. by Dr. Fekadu .T 8/27/2021 373

63-Fraon, a 10-month-old infant, has been unsettled and febrile with a runny nose for 2 days. Her family doctor examines her ear canal and the tympanic membrane appears as dull and bulging. below ,What is the most likely diagnosis? A. Acute otitis externa B. Cholesteatoma C. Chronic otitis externa D. Foreign body in the external ear canal E. Otitis media with effusion. prep. by Dr. Fekadu .T 8/27/2021 374

Ottitis Externa is diffuse inflammation of the external ear canal which may involve the pinna or the tympanic membrane. The most common causative agents being - Pseudomonas -Staphylococcus aureus and other -gram-negative microbes occurring as a polymicobial infection. -Fungal otitis externa can occur in the setting of repeated antibiotic use. - Frequent Swimming, Rugorous ear cleaning, excessive use of air phone,and underlying dermatological conditions can be risk factors for otitis externa . 8/27/2021 prep. by Dr. Fekadu .T 375

Acute otitis media is the rapid onset of signs and symptoms of inflammation of the middle ear cleft mostly following URTIs. The most common causative bacterial organism are Streptococus Pnemonia , Haemphilus Influenza A and Moraxella catarrhalis . Viral infection, commonest etiologies, may commonly prepare the way for secondary bacterial infection. 8/27/2021 prep. by Dr. Fekadu .T 376

Chronic otitis media is defined as long-standing inflammation of the middle ear cleft in which characterized by chronically discharging ears for > 12 weeks (3 months). Clinical Features Symptoms Constant or intermittent discharge (usually odorless) from the ear mostly not accompanied by otalgia Signs Hearing impairment in the affected ear. Otoscopy : Tympanic membrane Perforation-Tuning fork test ( Rhinee and Webers Test): Conductive Hearing Loss Symptoms of impending complications like fever, lethargy, headache, vomiting, neck pain, changing mentation , dizziness, vertigo). Investigations Culture of the ear discharge. Treatment Goals Keep the ear dry Eliminate the foci of infection in the temporal bone and the middle ear Construct the sound–conducting apparatus. Non pharmacologic Instruct patients to keep the ear dry (Vaseline gauze, dry it after showering) Aural toilet (recommended together with topical antibiotics) Pharmacologic N.B. In acute exacerbations only. Antibiotic treatment, whenever possible, must be directed by the results of culture and sensitivity of the ear discharge. First line Topical: Ciprofloxacin ear drop , 0.3%, 5ml. 2 – 3 drops twice daily for 02 weeks. N.B; Ciprofloxacin and other quinolone like Norfloxacin 8/27/2021 prep. by Dr. Fekadu .T 377

A. Acute otitis externa is an infection of the outer ear canal, not of the tympanic membrane. B. Cholesteatoma A rare problem in children. The two most common symptoms are persistent, often smelly, discharge from the affected ear and gradual loss of hearing in the affected ear. A cholesteatoma is an abnormal, non-cancerous skin growth that can develop in the middle section of your ear, behind the eardrum. It may be a birth defect, but it's most commonly caused by  repeated middle ear infections . A cholesteatoma often develops as a cyst, or sac, that sheds layers of old skin. C. Chronic otitis externa is an infection of the outer ear canal, not of the tympanic membrane.…history is only 2 days long and therefore not chronic. D. Foreign body in the external ear canal has symptoms of an upper respiratory tract infection and fever, and no foreign body is seen on examination. E. Otitis media with effusion Correct. Here the ear drum is dull and bulging. It is an accumulation of serous fluid in the middle ear Commonest cause of Conductive Hearing Loss in children Causes : Eustachian tube dysfunction prep. by Dr. Fekadu .T 8/27/2021 378

64-Abel , a 7-year-old boy, complains to his family doctor of a sore throat and has a mild fever. The appearance of his throat is shows intense inflammation of the tonsils with purulent exudates .What is the most likely diagnosis?. A. Diphtheria B. Glandular fever ( Ebstein Barr virus) C. Group A Streptococcal tonsillitis. D. Measles E. Herpes simplex stomatitis prep. by Dr. Fekadu .T 8/27/2021 379

A-Diphtheria is now extremely rare as children are immunized . Also the appearance is not ‘typical’. In diphtheria a thick grey pharyngeal membrane is characteristic. B. Glandular fever ( Ebstein Barr virus) Infection with Ebstein Barr virus can lead to tonsillitis although typically the tonsils are coated with a grey membrane. Palatal petechiae (pinpoint spots on the soft palate) may also be seen. C. Group A Streptococcal tonsillitis Correct. There is intense inflammation of the tonsils with purulent exudates. In this age group Group A beta- haemolytic Streptococcus is the most likely causative pathogen. D. Measles In measles there may be white spots ( Koplik spots) visible on the buccal mucosa. E. Herpes simplex stomatitis There are lesions on the lips, gums and tongue prep. by Dr. Fekadu .T 8/27/2021 380

65-Amen, a 9-month-old girl, presents with a 4-day history of coughing spasms which are followed by vomiting. Whooping cough ( Bordetella pertussis infection) is suspected. Which of the following tests would be most useful in confirming the diagnosis? Select one answer only. A. Blood culture B. Chest X-ray C. Full blood count and film D. Nasopharyngeal aspirate E. Pernasal swab , prep. by Dr. Fekadu .T 8/27/2021 381

Blood culture It is incredibly rare to isolate Bordetella pertussis from blood cultures. B. Chest X-ray Chest X-ray changes can occur in whooping cough but are not diagnostic. C. Full blood count and film A marked lymphocytosis is characteristic of pertussis , but not diagnostic. The lymphocytosis is secondary to pertussis toxin. Bordetella parapertussis does not produce pertussis toxin. D. Nasopharyngeal aspirate Immunofluorescence of a nasopharyngeal aspirate is used to identify respiratory syncytial virus, which causes bronchiolitis , but is not helpful in the diagnosis of pertussis . E. Pernasal swab Correct. Culturing a pernasal swab allows the pathogen ( Bordetella pertussis ) to be identified (though PCR (polymerase chain reaction) is more sensitive). This can also be helpful in isolating the related Bordetella parapertussis prep. by Dr. Fekadu .T 8/27/2021 382

66-A 3-year-old boy, presents to his family doctor. He has a ‘hacking’ cough that started several weeks ago and has failed to respond to two courses of antibiotics. He is otherwise well and has had no previous chest problems. On examination there is decreased air entry in the right lower zone with normal percussion note. His growth is normal. Which is the most appropriate next step? Select one answer only. A. Admit for intravenous antibiotic therapy B. Assess bronchodilator response C. Organize for ultrasound-guided drainage of his pleural effusion D. Request a chest X-ray. E. Request a sweat test and evaluation of immunoglobulins and functional antibodies prep. by Dr. Fekadu .T 8/27/2021 383

A-Admit for intravenous antibiotic therapy has already failed to respond to two coursesNof antibiotics, so further investigation is warranted. B. Assess bronchodilator response There is no wheeze on examination and so inhaled bronchodilator is unlikely to be effective. C. Organize for ultrasound-guided drainage of his pleural effusion.His signs are not consistent with a pleural effusion. The percussion note would be stony dull if this was a pleural effusion. prep. by Dr. Fekadu .T 8/27/2021 384

D. Request a chest X-ray The most likely diagnosis is the inhalation of a foreign body, e.g. a peanut. also has focal chest signs and so needs a chest X-ray to be performed. E. Request a sweat test and evaluation of immunoglobulins and functional antibodies A sweat test is used to diagnose cystic fibrosisand is not indicated here because this is his first episode of chest problems and he is growing normally. prep. by Dr. Fekadu .T 8/27/2021 385

67-Amir, a 4-year-old boy, presents to his family doctor with a history of eczema, rhinitis, chronic nocturnal cough and intermittent wheeze. Asthma is suspected and a bronchodilator is prescribed. How should his bronchodilator be delivered? Select one answer only. A. Dry powder inhaler B. Metered dose inhaler (MDI) C. Metered dose inhaler with large-volume spacer. D. Nebulizer E. Syrup prep. by Dr. Fekadu .T 8/27/2021 386

A-Dry powder inhaler Dry powder inhaler is appropriate only if MDI and spacer have failed in this age group. Most 4 year olds will perform better with MDI and spacer. B. Metered dose inhaler (MDI) Children should be prescribed a MDI with spacer as they cannot co-ordinate an MDI alone. It is always preferable to use a spacer, even in adults as delivery is more reliable. C. Metered dose inhaler with large-volume spacer Correct. The best mode of delivery is direct to the lungs. Children under 5 years of age should be prescribed a MDI with spacer as they cannot co-ordinate an MDI alone. D. Nebulizer Nebulizers provide very effective delivery of bronchodilators. However, they produce more hypoxia than bronchodilators delivered by pressurized MDI and spacer, are more expensive, and are not as safe. In hospital the nebulizers are driven by oxygen to offset the risk. E. Syrup should not be used as it is results in high blood levels and unacceptable side-effects. prep. by Dr. Fekadu .T 8/27/2021 387

68-Sara, a 10-year-old girl, has frequent attacks of asthma. She attends the EopD with increasing difficulty in breathing over the last 12 hours. Initial observation shows that she is anxious, sitting upright, has a marked tracheal tug and is unable to complete a sentence. Which of the following statements is most likely to be correct? Select one answer only. A. Sarah’s asthma attack is of moderate severity B. Sarah’s condition is likely to improve if she is encouraged to lie flat C. Sarah’s oxygen saturation should be measured. D. Sarah should be taken promptly to the X-ray department for a chest X-ray E. The lack of wheeze should make you consider a panic attack prep. by Dr. Fekadu .T 8/27/2021 388

A-Sarah’s asthma attack is of moderate severity Sarah’s attack is severe as she is unable to complete a sentence. B. Sarah’s condition is likely to improve if she is encouraged to lie flat Sitting upright assists lung mechanics and enables her to use her accessory muscles. C. Sarah’s oxygen saturation should be measured Correct. Oxygen saturation should be measured to further assess the severity of the asthma attack and to guide treatment. D. Sarah should be taken promptly to the X-ray department for a chest X-ray The priority is to treat Sarah’s asthma. If a chest X-ray is indicated, then a portable X-ray machine should be brought to any patient who is significantly unwell. E. The lack of wheeze should make you consider a panic attack The absence of wheeze implies that little air is moving in and out of the chest and would indicate severe bronchoconstriction . prep. by Dr. Fekadu .T 8/27/2021 389

69-Norah, an 18-month-old girl, presents to her family doctor with coryza , cough and a mild fever for 3 days. She feeds poorly and is unsettled at night. Her respiratory rate is normal and there is no chest recession. What is the most likely diagnosis? A. Bronchiolitis B. Frontal sinusitis C. Pneumonia (lower respiratory tract infection) D. Tonsillitis E. Upper respiratory tract infection , prep. by Dr. Fekadu .T 8/27/2021 390

A- Bronchiolitis Norah’s chest has no abnormal signs on auscultation. With bronchiolitis , you would expect to hear crepitations and wheeze, and to see signs of respiratory distress. B. Frontal sinusitis This is unusual in children of this age because frontal sinuses do not develop until late childhood. C. Pneumonia (lower respiratory tract infection) Young children with pneumonia can sometimes present without chest signs on auscultation, but here would be respiratory distress including a raised respiratory rate. D. Tonsillitis Examination of the oropharynx would show intense inflammation of the tonsils, often with a purulent exudate . E. Upper respiratory tract infection Correct. This child has an upper respiratory tract infection, most likely a common cold prep. by Dr. Fekadu .T 8/27/2021 391

70- A 5 month old infant is on formula feeding due to poor breast milk production by the mother on assessment the infant is found to have severe wasting wit weight to height is <3sd. What is the most appropriate intervention for the infant? A-start feeding with F-75 B-Start feeding with diluted F-100. C-Use supplementary sucking feeding D-give vit A 100.000IU At admission prep. by Dr. Fekadu .T 8/27/2021 392

Internal Medicine PART -III MCQ prep. by Dr. Fekadu .T 8/27/2021 393

1-What is the biological hallmark of Hyperuricemia in gout patients? A.Increased uric acid excretion or uric acid overproduction or both. B.Hyper production of uric acid or reduced uric acid excretion or both. C.Reduced uric acid production or decreased uric acid excretion or both. D.Idiopathic origins. prep. by Dr. Fekadu .T 8/27/2021 394

2-A Twenty eight-year-old patient has recently been diagnosed with Diabetes Mellitus. After a week, the patient complains progressive rise in blood glucose from bed time to morning. Which of the following characteristics of morning hyperglycemia is described? A-Dawn phenomenon B-Hypoglycemia C-Insulin waning D- Somogyi effect prep. by Dr. Fekadu .T 8/27/2021 395

Key: C -Waning insulin : -If   your insulin level falls too low overnight , your blood sugar rises {hyperglycemia}.The reasons for the drop in insulin vary from person to person, but it most commonly occurs when your insulin pump settings provide too little basal (background) insulin overnight or if your long-acting insulin dose is too low. B-Hypoglycemia C-The dawn phenomenon, also called the dawn effect, is the term used to describe  an abnormal early-morning increase in blood sugar (glucose)  — usually between 2 a.m. and 8 a.m. — in people with diabetes. D-The Somogyi effect or phenomenon happens  when you take insulin before bed and wake up with high blood sugar levels . According to the theory of the Somogyi effect, when insulin lowers your blood sugar too much, it can trigger a release of hormones that send your blood sugar levels into a rebound high. prep. by Dr. Fekadu .T 8/27/2021 396

3-A 25 year-old man with known type I DM is brought to a hospital. He is also a known hypertensive patient. His blood glucose level is 800 mg/dl, urine keton 3+, and PCO 2 is 10mmHg. Which one of is the following fluid is most appropriate to be given to this patient? A-0.45% sodium chloride solution B-0.9% sodium chloride solution C-DNS solution D-Dextrose 5% solutionj prep. by Dr. Fekadu .T 8/27/2021 397

4-Which of the following is an example of type IV hypersensitivity reactions? A-Allergic rhinitis B-Asthma C-Atopic dermatitis D-Contact dermatitis prep. by Dr. Fekadu .T 8/27/2021 398

ACID 1-Type I hypersensitivity reactions involve immunoglobulin E ( IgE ) antibody against soluble antigen, triggering mast cell degranulation . - Allergic example local and systemic aphylaxis , Food allergy ,drug allergy ,asthma, allergic: , rhinitis, conjunctivitis, and dermatitis), and allergic diseases, which are immune responses to foreign allergens ( i.e , urticaria , angioedema, ) 2-Type II hypersensitivity reactions involve IgG and IgM antibodies directed against cellular antigens, leading to cell damage mediated by other immune system effectors . - cytotoxic ex hemoytic animia 3-Type III hypersensitivity reactions involve the interactions of IgG, IgM, and, occasionally, IgA1 antibodies with antigen to form immune complexes. Accumulation of immune complexes in tissue leads to tissue damage mediated by other immune system effectors . - immune mediated reaction ex >serum sickness,PSGN , RA.SLE 4-Type IV hypersensitivity reactions are T-cell–mediated reactions that can involve tissue damage mediated by activated macrophages and cytotoxic T cells. Delayed EX Contact dermatitis,T1 DM Atopic dermatitis .*eczema prep. by Dr. Fekadu .T 8/27/2021 399

prep. by Dr. Fekadu .T 8/27/2021 400

5-One of your patients is awaiting lab results for kidney function. The patient has recently recovered from a streptococcal throat infection. Which one of the following kidney disorders might the patient have? A-Chronic renal failure B- Glomerulonephritis C-Acute renal failure D- Pyelonephritis prep. by Dr. Fekadu .T 8/27/2021 401

6-Which of the following is the first line drug of choice for a hospital acquired gram negative bacterial pneumonia? A-Metronidazole500mg potid B-Amoxicillin 500mg po / iv tid C- Gentamycin 80mg iv / imtid D-Cloxacillin500mg po / iv qid prep. by Dr. Fekadu .T 8/27/2021 402

7-A -24- year old woman is brought to a hospital with a diagnosis of blood disorder. Onlaboratory analysis hematocrit become 55%.Which type of blood disorder will explain it? A- Hemolyticanemia B-Iron deficiency anemia C- Aplastic anemia D- Polycythemia prep. by Dr. Fekadu .T 8/27/2021 403

Polycythemia Hematocrit — The hematocrit (HCT) is expressed as the percent of a blood sample occupied by intact RBCs. Polycythemia in the adult patient is suspected when the HCT is >48 or >52 percent in women and men, respectively. Hemoglobin concentration — The hemoglobin concentration (HGB) is its content in grams per 100 mL of whole blood. Polycythemia in the adult is suspected when the HGB is >16.5 or >18.5 g/ dL  in women and men, respectively. prep. by Dr. Fekadu .T 8/27/2021 404

8-Which of the following is not a route of hepatitis B transmission? A-Blood contact B-Sexual contact C-Animal contact D- Perinatal transmission prep. by Dr. Fekadu .T 8/27/2021 405

9-What is the first thing you should do for a patient having epileptic seizure? A-Give diazepam B-Prevent tong biting by inserting oral air way C-Put him or her on the lateral position D-Restrains the patient prep. by Dr. Fekadu .T 8/27/2021 406

10-What would be the first line emergency management of a patient with acute asthma? A- Aminophyline B-Hydrocortisone C-Oxygen D- Salbutamol prep. by Dr. Fekadu .T 8/27/2021 407

11-A -32- year old known RVI patient who has diarrhea for the past 1month with decrease intake came to the ED with weakness and change in mentation of 1 day duration. On physical examination, Bp- un recordable, PR 140 b/m, RR 36 b/m, T o 35 o c, Oxygen saturation- 80%. What is your priority management for this patient? A-Check his airway, breathing and secure wide bore bilateral IV line B-Secure bilateral IV line C-Start warming up the patient because he is hypothermic D-Take further history regarding the duration and the quality of the diarrhea prep. by Dr. Fekadu .T 8/27/2021 408

12-A -20-year old male patient who sustained car accident came to you with loss of consciousness of 5 hours duration. On P/E, the patient is snoring with excessive secretions from his mouth. What will be the priority nursing management for this patient? A-Take vital signs B-Assess for presence of bleeding C-Open airway and apply suctioning D-Secure bilateral IV lines to manage shock prep. by Dr. Fekadu .T 8/27/2021 409

13-W/ ro Hanna is a known Chronic kidney disease patient with urine output of <300ml per day. How do you report W/ ro Hanna’s finding? A- Anuria B- Nocturia C- Oliguria D- Polyuria prep. by Dr. Fekadu .T 8/27/2021 410

Oliguria  is defined as a urine output that is less than 1 mL/kg/h in infants, less than 0.5 mL/kg/h in children, and less than 400 mL daily  . Anuria  or anuresis occurs when the kidneys aren't producing urine. A person may first experience oliguria, or low output of urine, and then progress to  ... Polyuria  is urine output of > 3 L/day; it must be distinguished from urinary frequency, which is the need to urinate many times during the day or night but  prep. by Dr. Fekadu .T 8/27/2021 411

14-What is the priority nursing intervention for a patient who has hypotension secondary to diarrhea and vomiting? A-Give medication B-Identify underline cause C-Resuscitate with fluid D-Taking detailed history prep. by Dr. Fekadu .T 8/27/2021 412

16-What is the correct compression to ventilation ratio of an adult patient receiving CPR, respectively? A-15:2 B-2:30 C-30:2 D-60:2 prep. by Dr. Fekadu .T 8/27/2021 413

17-Which of the following is true about MDR TB? A-Resistance to Ethambutol and Isonaizide B-Resistance to Isonaizide C-Resistance to Refampcin D-Resistance to Refampcin and Isonazide prep. by Dr. Fekadu .T 8/27/2021 414

18-Which one of the following statement is true about type 1 DM? A-DKA is less likely than type 2 DM B-More common in old age than type 2 DM C-Necessarily needs insulin therapy D-Nutritional management is mandatory than type 2 prep. by Dr. Fekadu .T 8/27/2021 415

19-Which one of the following investigation is a golden standard test for TB? A.X-ray B.AFB C.CT D.Culture prep. by Dr. Fekadu .T 8/27/2021 416

20-W/ roAbebech comes to your institution within an hour of ingestion of rat poison. What will be the emergency management of this patient? A-Cleansing Enema B-Decompression C-Gastric Gavage D-Gastric Lavage prep. by Dr. Fekadu .T 8/27/2021 417

21-Which one of the following is positive clinical finding of meningitis? A- Brudzinski sign B-Fever C-Headache D-Vomiting prep. by Dr. Fekadu .T 8/27/2021 418

prep. by Dr. Fekadu .T 8/27/2021 419

22-Which one of the following is the major criterion for diagnosing acute rheumatic fever? A- Arthlargia B- Carditis C-Fever D-Lab investigation prep. by Dr. Fekadu .T 8/27/2021 420

prep. by Dr. Fekadu .T 8/27/2021 421

prep. by Dr. Fekadu .T 8/27/2021 422

23-Which one of the following is true about hypertensive emergency? A-Bp>180/120mmhg and no organ failure B-Bp>180/120mmhg and organ failure C-Organ failure D-Severe rise in blood pressure (Bp>180/120mmhg) prep. by Dr. Fekadu .T 8/27/2021 423

A hypertensive emergency A hypertensive emergency is the association of substantially elevated BP with acute Hypertension Mediated Organ Damage (acute HMOD). Target organs include the retina, brain , heart, large arteries, and the kidneys. Examples include malignant hypertension, with or without hypertensive encephalopathy, subarachnoid or intracerebral hemorrhage, acute pulmonary edema, aortic dissection, and rebound after withdrawal of antihypertensive medications . Tx 1 st line Labetalol 20mg iv over 2 min initially then 40_80mg iv q10min not exceed 300mg alternative Hydralazine 10 mg po QID For 4 DAYS or 20 -40 mg IV Or IM Hypertensive urgency   — Severe hypertension (as defined by a diastolic blood pressure above 120 mmHg) in asymptomatic patients is referred to as hypertensive urgency. There is no proven benefit from rapid reduction in BP in asymptomatic patients who have no evidence of acute end-organ damage and are at little short-term risk prep. by Dr. Fekadu .T 8/27/2021 424

Hypertensive Crises ( BP> 180/110 mmHg) with or without target organ damage Diagnosing hypertension The diagnosis of hypertension should be confirmed at an additional patient visit, usually 1 to 4 weeks after the first measurement depending on the measured values and other circumstances. In general, hypertension is diagnosed if, on two visits, on different days: Systolic blood pressure on both days is ≥140 mmHg and/or diastolic blood pressure on both days is ≥90 mmHg. Once diagnosis of hypertension is made: Look for end-organ damage based on: prep. by Dr. Fekadu .T 8/27/2021 425

prep. by Dr. Fekadu .T 8/27/2021 426

Look for end-organ damage based on: History : Symptoms of heart failure (SOB, unusual fatigue and body swelling), history of sudden onset body weakness (stroke), intermittent claudication institutions, severe headache and blurring of vision. Physical Examination: Pulse rate and rhythm, signs of heart failure (edema, elevated JVP, crackles on the lungs), Focal neurologic deficit, eye signs. The physical examination should be done to the maximum capacity of the health work force including fundoscopic retinal examination if possible . prep. by Dr. Fekadu .T 8/27/2021 427

INVX Laboratory and other diagnostic tests: Health facilities should thrive to avail at east mandatory tests. Please note that waiting for laboratory tests shouldn‘t delay the intervention of hypertension as the disease do much harm than the extra benefit obtained from the tests. The tests are categorized as follows : Mandatory tests at diagnosis (urine dipstick to check for protein , Creatinine to check for renal function) Optional tests at diagnosis (ECG to look for effect of blood pressure on the heart, Serum electrolytes mainly potassium, Thyroid function test to assess a secondary cause of hypertension ) Indication based tests (Echocardiography for heart failure patients, brain imaging for suspicion of stroke) Comorbidity and risk factor assessment tests (Blood sugar and cholesterol ) Look for risk factors: oHistory : Smoking, excess salt intake, sedentary life, low fruit and vegetable intake, excess alcohol consumption prep. by Dr. Fekadu .T 8/27/2021 428

Hypertension Treatment Who should receive hypertension treatment? Hypertension treatment is indicated for adults diagnosed with hypertension, as defined (SBP ≥140 mmHg and/ Or DBP ≥90 mmHg). Patients with SBP ≥180 mmHg or DBP ≥ 110 mmHg may be indicated for immediate treatment based on one assessment. prep. by Dr. Fekadu .T 8/27/2021 429

Lifestyle counseling (healthy diet, physical activity, tobacco use, and harmful use of alcohol ) is a critical component of good hypertension management and is often recommended as a first step for patients with blood pressure of SBP 130–139 mmHg and/or DBP 80–89 mmHg who do not have other CVD risk factors. However , in settings where people do not regularly visit the doctor, people who are recommended only lifestyle modification may not return for re-evaluation and needed treatment, resulting in uncontrolled hypertension and associated complications . Non pharmacologic (Life Style interventions) All patients diagnosed to have hypertension should be given lifestyle interventions which include: o Consume healthy diet: o Engage in physical activity, o Avoid smoking o Limit use of alcohol prep. by Dr. Fekadu .T 8/27/2021 430

Pharmacologic (Drug Treatment) Indications: Patient who couldn‘t achieve target blood pressure with life style interventions . At initial presentation in those with: End-organ damage or high WHO cardiovascular risk (>20%) Hypertensive Crises prep. by Dr. Fekadu .T 8/27/2021 431

Treatment targets For most patients, blood pressure is considered controlled when SBP <140 mmHg and DBP <90 mmHg. Life style intervention Life style intervention should be implemented in all grades of hypertension. For uncomplicated grade 1 hypertension life style intervention can be tried for three months before initiation of medications. If failed to achieve a blood pressure of less than 140/90 mmHg , then initiation of antihypertensive medication is recommended . Initial monotherapy in uncomplicated hypertension : Long-acting dihydropyridine calcium channel blocker such as amlodipine as first line drug for the treatment of uncomplicated essential hypertension Dose: Amlodipine 5 mg daily, escalate to 10 mg if BP is uncontrolled Thiazide diuretics such as hydrochlorothiazide to be used as add on when target BP not achieved on long-acting dihydropyridine calcium channel blocker such as amlodipine Dose: Hydrochlorothiazide 25 mg Po daily prep. by Dr. Fekadu .T 8/27/2021 432

If a third agent is needed, the alternative class of medication is ACE inhibitors considering cost of the drug and availability at the General Hospitals in Ethiopia. Lisinopril is the preferred drug in this class due to its ease of administration. The alternative is Enalapril . Dose: Lisinopril 5 mg daily, escalate dose to 40 mg Po daily . if BP is uncontrolled Enalapril 5 mg Po BID , escalate to 20 mg Po BID if BP is uncontrolled prep. by Dr. Fekadu .T 8/27/2021 433

Notes on specific hypertension medications Pregnant women and women of childbearing age not on effective contraception should not be given ACE inhibitors, ARBs, or thiazide/thiazide-like diuretics; CCBs should be used . I.E nifedipine . Amlodipine , felodipine,iserodipine . If not controlled with intensification dose of medication, refer to specialist. Beta blockers are not recommended as first-line therapy. If a heart attack has been diagnosed within the previous three years, or there is atrial fibrillation or heart failure, then a beta blocker should be added to the starting dose of antihypertensive medication. Patients with angina may also benefit from treatment with a beta blocker. prep. by Dr. Fekadu .T 8/27/2021 434

24-Which of the following is isotonic solution? A-3% Normal saline B-5% Normal saline C-0.9% Normal saline D-0.45% Normal saline prep. by Dr. Fekadu .T 8/27/2021 435

Isotonic solutions have the same osmotic pressure as plasma, creating constant pressure inside and outside the cells, which causes the cells to remain the same (they will not shrink or swell) and does not cause any fluid shifts within compartments. Isotonic solutions are useful to increase intravascular volume, and are utilized to treat vomiting, diarrhea, shock, and metabolic acidosis, and for resuscitation purposes and the administration of blood and blood products . Examples of isotonic solutions include normal saline (0.9% sodium chloride), lactated Ringer’s solution, 5% dextrose in water ( D5W ), and Ringer’s solution. prep. by Dr. Fekadu .T 8/27/2021 436

Hypertonic solutions  have a higher concentration, or tonicity, of solutes and have an osomolality equal to or greater than 375 mOsm /L . The osmotic pressure gradient draws water out of the intracellular space into the extracellular space. Examples of hypertonic solutions include D5W and 0.45% sodium chloride, D10W, and 3% sodium chloride. Hypertonic solutions may cause intravascular fluid volume overload and pulmonary edema, and they should not be used for an extended period of time. Hypertonic solutions should not be used in patients with heart or renal disease who are dehydrated prep. by Dr. Fekadu .T 8/27/2021 437

Hypotonic solutions  have a lower concentration, or tonicity, of solutes and have an osomolality equal to or less than 250 mOsm /L. The infusion of hypotonic solutions lowers the osmolality within the vascular space and causes fluid to shift to the intracellular and interstitial space. Cells will swell but may also delete fluid within the vascular space . Examples of hypotonic solutions include 0.45% sodium chloride, 0.33% sodium chloride, 2.5% dextrose in water, and 0.2% sodium chloride. Monitor for hypovolemia and hypotension related to fluid shifting out of the vascular space, and do not administer to patients with increased intracranial pressure (ICP), as it may exacerbate cerebral edema. Use cautiously in patients with burns, liver failure, and traumas prep. by Dr. Fekadu .T 8/27/2021 438

25-Which one of the following is the best manifestation of right side heart failure? A-Cough B-Decreased urine production C-Difficulty of laying flat D-Jugular vein distention prep. by Dr. Fekadu .T 8/27/2021 439

Right-sided heart failure generally  develops as a result of advanced left-sided heart failure , Cor Pulmonale (Right-sided heart failure) The most common cause of right-sided heart failure is actually  left-sided heart failure . But other conditions, such as certain lung diseases, can cause the right ventricle to fail even when there is no problem with your left ventricle. prep. by Dr. Fekadu .T 8/27/2021 440

Right-sided heart failure  occurs in about one in 20 cases of heart failure, often following left-sided heart failure . Many of the symptoms are related to fluid retention in various organs and tissues. In particular, congestion in peripheral capillary beds causes swelling under the skin (peripheral edema) that may move around according to the forces of gravity. For example, feet and ankles may swell when you are standing up, while areas of lower back may swell when lying down (known as sacral edema ). Peripheral edema in legs may then lead to nocturia , the need to urinate frequently during the night. This happens because fluid retained in legs during the day flows back into bloodstream when lay down, and is processed into urine in kidneys while sleep . In more severe cases, the edema caused by right-sided heart failure, may result in pitting peripheral edema, a swollen abdomen due to fluid retention in the space within the abdominal cavity (ascites), or liver enlargement due to fluid Retention/ prep. by Dr. Fekadu .T 8/27/2021 441

prep. by Dr. Fekadu .T 8/27/2021 442

26-What makes the pain of myocardial infarction different from that of angina pectoris? A-The pain is localized B-The pain is sever and radiating C-Pain is relieved by rest D-The pain begin after vigorous exercise prep. by Dr. Fekadu .T 8/27/2021 443

27-Which one of the following is true about semi- fowler position of a patient with congestive heart failure? A-Reduce venous return from the lower body B-Enhance renal perfusion C-Enhance digestion D-Reduce jugular venous pressure prep. by Dr. Fekadu .T 8/27/2021 444

prep. by Dr. Fekadu .T 8/27/2021 445

prep. by Dr. Fekadu .T 8/27/2021 446

29-A 24- year- old patient is brought to a hospital with a clinical manifestation of Dyspepsia, discomfort on the epigastrium and burning sensation in the esophageal area. Upon Laboratory examination,Serologic test revealed positive for H. pylori Ag. Which of the following agent will not treat the bacteria? A- Amoxacillin B- Claritromycin C- Metronidazole D- Omeprazole prep. by Dr. Fekadu .T 8/27/2021 447

30-Which of the following manifestation describes severe cerebral malaria? A-Fever B-Chills C-Shiver D-Hypoglycemia prep. by Dr. Fekadu .T 8/27/2021 448

Hypoglycemia: A common complication of malaria and a marker of severe disease; Suspecte in any patient who deteriorates suddenly. Threshold for intervention among children <5 years is <3 mmol /L (<54 mg/ dL ); children ≥5years and adults is <2.2 mmol /L (<40mg/ dL ). Hypoglycemics need intravenous access established promptly, followed by administration of initial bolus of dextrose (0.25 g/kg of body weight ), may be infused over a period of 3-5minutes, which is usually achieved with 2.5 mL /kg of 10 percent dextrose solution. Blood glucose measurement after 15 minutes should be repeated, with administration of repeat boluses until the patient is normoglycemic . Re-check blood glucose every 2-4 hrs during course of treatment, particularly in pregnant or comatose patient because hypoglycemia can recur even after an IV bolus of glucose prep. by Dr. Fekadu .T 8/27/2021 449

Hematologic complications (including severe anemia and coagulopathy ) Anemia: If haemoatcrit is below 15% (Hg <5g/dl) in a normally hydrated child or adult, a blood transfusion is indicated: 10 ml of packed cells OR 20 ml whole blood/kg of body weight. Bleeding tendency: Check bleeding time of the patient, crossmatch blood, give whole fresh blood or platelet infusion as needed to correct blood loss and bleeding. hypovolaemia and acidosis Shock:  Management: maintain fluid balance, administer 20ml/kg fluid bolus, and check for bacteremia (blood cultures, WBC) give appropriate antibiotic, monitor vital signs. Acute renal failure:  Persistent oliguria (<17 ml/hour in adults: 0.3 ml/kg/hour in children) despite adequate correction of dehydration or hypotension. Rare in children than adults. Pulmonary complications: Pulmonary edema and Adult Respiratory Distress Syndrome (ARDS) Neurologic complications (include altered sensorium , seizure, and coma): prep. by Dr. Fekadu .T 8/27/2021 450
Tags